175
MODUL KERTAS 1 & 2 MODUL 1

BIO P1,P2 (2)

Embed Size (px)

DESCRIPTION

nice

Citation preview

Page 1: BIO P1,P2 (2)

MODUL KERTAS 1 & 2

MODUL 1

Page 2: BIO P1,P2 (2)

4551/1 LONJAKAN SAUJANA SPM 2009

PPD MELAKA TENGAH

2

1 Diagram 1 shows a type of muscle tissue found in the human body. Where is thetissue found?

Diagram 1

A Heart C BicepsB Pancreas D Small intestine

2 The plasma membrane consists of molecules arranged in a double layer asshown in Diagram 2

Diagram 2

The part labelled I and II areA hydrophobic and hydrophilic respectivelyB hydrophilic and hydrophobic respectivelyC both hydrophobicD both hydrophilic

3. Diagram 3 shows the net flow of water molecules from a dilute solutionto a concentrated solution through a semi-permeable membrane.

Diagram 3

What is this process called?A OsmosisB Active transportC Simple diffusionD Facilitated diffusion

Page 3: BIO P1,P2 (2)

4551/1 LONJAKAN SAUJANA SPM 2009

PPD MELAKA TENGAH

3

4 Diagram 4 is a graph which shows the changes in mass of potato strips immersed indifferent concentrations of sucrose solution.

Diagram 4

Based on the graph, which of the following concentrations of sucrose solution shouldbe used so that a flaccid potato strip regains its turgidity?A 1.5 g per 100 mlB 2.5 g per 100 mlC 3.5 g per 100 mlD 4.5 g per 100 ml

5. Lipid is needed to build substance X while cellulose is needed to build substance Y.What are substances X and Y?

Substance X Substance Y

A Protoplasm Cell wall

B Plasma membrane Cell wall

C Cell wall Protoplasm

D Cell wall Plasma membrane

6 . Haemoglobin is an example of aA. primary structure of proteinB. secondary structure of proteinC. tertiary structure of proteinD. quarternary structure of protein

Page 4: BIO P1,P2 (2)

4551/1 LONJAKAN SAUJANA SPM 2009

PPD MELAKA TENGAH

4

7 Which of the following graph shows the effect of pH on the metabolic rate of theenzyme pepsin?

8 Diagram 5 shows the phases in a cell cycle.

Diagram 5

Yphase

X phase

Rate ofreaction Rate of

reaction

Rate ofreaction Rate of

reaction

X phase

Page 5: BIO P1,P2 (2)

4551/1 LONJAKAN SAUJANA SPM 2009

PPD MELAKA TENGAH

5

Which of the following represents V ?

A mitosis C stage SB cytokinesis D stage G1

9 Diagram 6 shows a phase in mitosis of a plant cell.

Diagram 6

Which of the following is true about the cell in Diagram 6 ?

Stage of mitosis Number of chromosomes in themother cell

A Anaphase 4B Telophase 4C Anaphase 8D Telophase 8

10. Crossing over is an important process in meiosis. It results in variations in the daughtercells. At which stage of meiosis does crossing over take place?

A Prophase IB Prophase IIC Metaphase IID Anaphase I

11 Lack of vitamin D in the diet will cause the disease

A scurvy.B beri-beri.C rickets.D pellagra.

Page 6: BIO P1,P2 (2)

4551/1 LONJAKAN SAUJANA SPM 2009

PPD MELAKA TENGAH

6

12 Diagram 7 shows the molecular structure of three food classes

Diagram 7

Which food classes do X, Y and Z belong to ?

X Y ZA CarbohydrateB ProteinC ProteinD Lipid

ProteinLipid

CarbohydrateCarbohydrate

LipidCarbohydrate

LipidProtein

13 Diagram 8 shows part of the human alimentary canal.

Diagram 8

A person who has structure P removed must regulate his dietary intake of

A. fats C proteinB. carbohydrate D. water

14 In the absence of oxygen the skeletal muscles contract using energy from thebreakdown of glucose and glycogen toA ethanol and waterB energy and waterC ethanol and lactic acidD lactic acid and energy

Page 7: BIO P1,P2 (2)

4551/1 LONJAKAN SAUJANA SPM 2009

PPD MELAKA TENGAH

7

15 Which of the following is not involved in the transportation of carbon dioxide by theblood.A Carbonic acidB CarbaminohaemoglobinC Hydrogen carbonate ionD Carbon monoxide

16 Table shows the biomass of a few types of organisms in a community.

Organism Total Biomass of all theorganisms at eachtrophic level(kg)

P 1200Q 30R 150S 670T 100U 2700

Which of the following is a possible food chain in this community?

A P → U → S → T C P → S → T → QB Q → R → S → P D U → S → P → Q

17 Which of the following organisms is a parasite ?

A C

B D

Page 8: BIO P1,P2 (2)

4551/1 LONJAKAN SAUJANA SPM 2009

PPD MELAKA TENGAH

8

18 Diagram 9 is a graph which shows changes in the population of two species ofbeetles, T and U, over a period of time. Both species feed on the same food source.

Diagram 9

What is the interrelationship between species T and U?

A Symbiosis C ParasitismB Predation D Competition

19 Diagram 10 shows a cross section of a young dicotyledon root consisting of a fewmain tissues.

Diagram 10

What is Y ?A Phloem B XylemC Cortex D Cambium

20 Diagram 11 shows some human bones .

Diagram 11

Population

T U

Page 9: BIO P1,P2 (2)

4551/1 LONJAKAN SAUJANA SPM 2009

PPD MELAKA TENGAH

9

Which bones are part of the axial skeleton ?A P and SB Q and RC P , Q and RD Q , R and S

21 Diagram 12 below shows a motor neuron.

Diagram 12

Which one of the following A, B, C or D in the table below names the labelled partscorrectly?

1 2ABCD

Cell bodyAxonDendriteSynapse

AxonDendriteMyelin sheathDendrite

22 Diagram 13 shows a plant with soft stem .

Which of the following support structures helps the plant climb to obtain sunlight ?

Diagram 13

A Clasping roots B Twining stemsC Tendrils D Thorns.

Page 10: BIO P1,P2 (2)

4551/1 LONJAKAN SAUJANA SPM 2009

PPD MELAKA TENGAH

10

23 Diagram 14 shows a nerve pathway involved in a reflex action. Which structure is theefferent neurone?

Diagram 14

24 Some cucumber slices are immersed in 0.1% sucrose solution. After 3 hours, theslices are found to be turgid and hard.Which of the following statements explains this phenomenon?

A The cucumber cell wall prevents it from shrinkingB The cell sap is hypotonic towards the sucrose solutionC The high concentration of the cell sap in the vacuole causes water to

diffuse into the cellD The cucumber cell wall allows the sucrose molecules to diffuse into the

cell

25 Diagram 15 shows the four-chambered stomach of a ruminant.

Diagram 15

Which of the following is not a correct match about each chamber and its function?

A. S- Rumen, mutualistic bacteria digest cellulose.B. V- Reticulum, the partially digested food is formed into balls and regurgitated into the

mouth for chewingC. U- Omasum, the regurgitated food in the mouth is passed into the omasumD. T- Duodenum, the digested products are absorbed.

Page 11: BIO P1,P2 (2)

4551/1 LONJAKAN SAUJANA SPM 2009

PPD MELAKA TENGAH

11

26 What substances are dissolved in the fuid which passes along the ureter to thebladder of a healthy person?

Glucose Protein Salts UreaA. Absent Absent Absent PresentB. Absent Absent Present PresentC. Present Absent Present PresentD. Present Present Absent Absent

27 Diagram 16 shows the female reproductive system. In which parts are the eggs andthe zygote formed?

Diagram 16

Eggs Zygote

A

B

C

D

1

1

2

2

2

3

1

3

28 Diagram 17 shows sex determination in human.

Ovary cell Testis cell

Gametes

Diagram 17

1

2

P Q

R

Page 12: BIO P1,P2 (2)

4551/1 LONJAKAN SAUJANA SPM 2009

PPD MELAKA TENGAH

12

If a couple have a son, what is the cell composition in P, Q and R?

P Q RA 44 + X 44 + Y 44 + XYB 22 + Y 22 + X 44 + XYC 22 + X 22 + X 44 + XXD 22 + X 22 + Y 44 + XY

29 Which of these genetically inherited disease is dangerous and can cause deathat a young age?

A. AlbinoB. HaemophiliaC. Short-sightednessD. Down syndrome

30 Table 1 shows a Punnet square which represents the gametes and progenyfrom a dihybrid cross. Alphabets a to p represent the daughter cells from thiscross.

male gamete

female gamete

HK Hk hK hk

HK a b c dHk e f g hhK I j k lhk m n o p

Table 1

Which of the following daughter cells have the same genotype as the parent ?

A a, f, k, p C d, g, j, mB b, c, e, I D e, f, h, l

31 Diagram 18 shows the development of a pollen tube and its entry into the ovule.Which part develops into the testa after fertilisation?

Diagram 18

Page 13: BIO P1,P2 (2)

4551/1 LONJAKAN SAUJANA SPM 2009

PPD MELAKA TENGAH

13

32 Contraceptive pills contain a combination ofA. estrogen and luteinising hormone.B. progesterone and prolactinC. estrogen and follicle stimulating hormoneD. progesterone and estrogen.

33. Which of the following shows the differences between mitosis and meiosis?

Mitosis MeiosisI Involves one stage of cell division Involves two stages of cell division

II Produces two diploid daughter cells Produces four haploid daughtercells

III Synapsis and crossing over takesplace between homologouschromosomes

Synapsis and crossing over doesnot take place

IV Chromosomes are not in pairs Homologous chromosomes are inpairs at prophase I

A I and II onlyB I and III onlyC I, II and IV onlyD I, II, III and IV

34 Diagram 19 is a graph which represents a type of variation found in students.

Diagram 19This variation may be

I height II weightIII type of ear lobe IV dimpleA I and II only C III and IV onlyB I and III only D I, II, III and IV

Variation

Number ofindividuals

Page 14: BIO P1,P2 (2)

4551/1 LONJAKAN SAUJANA SPM 2009

PPD MELAKA TENGAH

14

35 Which of the following is not a cause of variation?

A. Radiation B. Gene mutationC. Asexual reproduction D. Meiosis

36 Diagram 20 shows the structure of a villus in the ileum

Diagram 20

Which vessels P, Q, R, and S carry the largest amounts of glucose, amino acids, lipiddroplets or fat-soluble vitamins?

Glucose Aminoacids

Lipiddroplets

Fat solublevitamins

A P Q R S

B Q P R S

C R R S S

D S S R R

37 Diagram 21 shows a plastic quadrat used to determine the percentage coverage ofbread mould on a piece of bread. The shaded area shows the presence of mould.

Diagram 21

Based on the result obtained in Diagram 21, calculate the percentage coverage ofthe bread mould , taking the area of each small square to be 1 cm²

A 32 % C 56%B 40% D 80%

Page 15: BIO P1,P2 (2)

4551/1 LONJAKAN SAUJANA SPM 2009

PPD MELAKA TENGAH

15

38 Diagram 22 below shows equipment that can cause the thinning of the ozone layer.

Diagram 22

How can this problem be solved?

I Stop using chlorofluorocarbonII Replace CFCs with HCFCsIII Patching holes in the ozone layerIV Produce less electrical goods

A I and IIB II and IVC I, II and IIID I, II, III and IV

39 The following statement is about eutrophication.

What is the correct sequence of the eutrophication process ?

A O, L, M and N C L, M, O and NB L, N, M and O D O, M, L and N

L : Excess fertilisers from agriculture lands flow into lakesM : Bacteria grow rapidlyN : Algae grow rapidly and covers the surface of the lakeO : The value of BOD increase

Page 16: BIO P1,P2 (2)

4551/1 LONJAKAN SAUJANA SPM 2009

PPD MELAKA TENGAH

16

40. An experiment was carried out to investigate the rate of water loss from a plant in aday.The wind and relative humidity factors were kept constant. Diagram 23 is a graphwhich shows the result obtained from 0600 to 1300 hours .Which of the curves A,B,Cor D is expected to show the rate of water loss in the plant after 1300 hours ?

Diagram 23

41 Diagram 24 shows a human arm .

Diagram 24

If tendon X was torn off,what happens to the arm ?A The elbow joint loosens upB The fingers cannot gripC The arm connot be bentD The lower arm cannot twist.

Page 17: BIO P1,P2 (2)

4551/1 LONJAKAN SAUJANA SPM 2009

PPD MELAKA TENGAH

17

42 Diagram 25 is a graph which shows the changes in the glucose concentration in theblood of a person over a period of two hours.

Diagram 25

Which of the following best explains the shape of the graph after X?A. The person has eaten a meal that is high in sugar .B. The person has had an insulin injection.C. The person is suffering from diabetes mellitus.D. The person starts some vigorous physical exercise.

43 Which characteristics of the glomerulus enhances the efficiency of ultrafiltration ?I. The diameter of the afferent arteriole is larger than that of the efferent arteriole.II. The afferent arteriole divides further into a dense network of capillaries.

III. The high hydrostatic pressure of the blood entering the glomerulus.IV. The Bowman’s capsule is made up of only two layers of cells.

A. I, II and III onlyB. I, II and IV onlyC. II, III and IV onlyD. I, II, III and IV

44 A woman’s menstrual period started on 23rd March. In which week was an egg mostlikely to have been released?

Week MarchSun Mon Tue Wed Thu Fri Sat

A - - - 1 2 3 4B 5 6 7 8 9 10 11C 12 13 14 15 16 17 18D 19 20 21 22 23 24 25

26 27 28 29 30 31

Page 18: BIO P1,P2 (2)

4551/1 LONJAKAN SAUJANA SPM 2009

PPD MELAKA TENGAH

18

45 What is true about the importance of secondary growth in plants?I. It increases the diameters of the plant stems and roots for mechanical support.II. It allows plants to increase in length to achieve maximum height.III. It produces new phloem and xylem tissues to replace the old and damaged ones.IV. It produces a thick bark which reduces the evaporation of water from the surface

of the stem.

A. I, II and III only B. I, III and IV onlyC II, III and IV only D. I, II, III and IV.

46 A woman with blood group A claims that a man with blood group AB is thefather of her baby. The baby’s blood is tested. Which of the following could be thebaby’s blood group?

I Group AII Group BIII Group OIV Group AB

A I and II onlyB I and IV onlyC I, II and IV onlyD I, II, III and IV

47 Which of the following shows the difference between continuous variation anddiscontinuous variation?

Continuous Variation Discontinuous VariationA Controlled by dominant genes. Controlled by recessive genes.B Caused by mutation. Not caused by mutation.C Occurs in animals. Occurs in plants.D Can be measured. Cannot be measured.

48 The Hydrangea plant produces blue flowers when grown on acidic soil, and redflowers when grown on alkaline soil. What conclusion can be made from thisobservation?

A. The colour of the Hydrangea flower is a continuous variationB. The environment affects the colour of the flowersC. The pH of the soil causes mutationD. The colour of the flower is affected by the genetic factor only

Page 19: BIO P1,P2 (2)

4551/1 LONJAKAN SAUJANA SPM 2009

PPD MELAKA TENGAH

19

49 I - Homologous chromosomes line up at the equator of the cell.II - Homologous chromosomes separate and move to opposite polesIII – Sister chromatids separate and move to different polesIV – Nuclear membrane and nucleolus disintegrate.

I, II, III and IV shows the processes which occur in meiosis. Among the followingevents, which occur in anaphase I ?.

A I onlyB II onlyC III and IV onlyD II and IV only

50 Diagram 26 shows an organ system.

Diagram 26

What are the functions of the organ system shown above?I To transport oxygen to the body cellsII To defend the body against diseasesIII To remove metabolic wastesIV To help regulate the volume and composition of bloodA I and II onlyB I and III onlyC II and IV onlyD III and IV only

END OF QUESTION PAPERKERTAS SOALAN TAMAT

Page 20: BIO P1,P2 (2)

4551/2 - 2 - LONJAKAN SAUJANA SPM 2009

PPD MELAKA TENGAH

Section A[ 60 marks ]

Answer all questions from this section.

1. Diagram 1 shows cell organisation in plant. Cells J undergo differentiation andspecialisation to form several tissues in a leaf of a green plant.Rajah 1 menunjukkan organisasi sel dalam satu tumbuhan. Sel-sel J mengalamipembezaan dan pengkhususan untuk membentuk beberapa tisu dalamsatutumbuhan hijau.

Cell Specialisation

DIAGRAM 1

(a) Name tissue K and tissue L.Namakan tisu K dan L

K : ………………………………………………………………………………………

L: …………………………………………………………………………………………

[2 marks]

Cells JSel-sel J

Cross-sectionof a leafKeratanrentas sehelaidaun

K

L

XylemtissueTisuxilem

M

Page 21: BIO P1,P2 (2)

4551/2 - 3 - LONJAKAN SAUJANA SPM 2009

PPD MELAKA TENGAH

(b) State the function of cells K and M in a leaf.Nyatakan fungsi sel K dan M dalam sehelai daun

K : ……………………………………………………………………………………….

M: ……...……………………………………………………………………………………

[2 marks]

(c) (i) Explain the differentiation of cells J to form the xylem tissue.Terangkan pembezaan sel J dalam membentuk tisu xilem.

……………………………………………………………………………………………………

……………………………………………………………………………………………………

[2 marks]

(ii) During the formation of the xylem tissue, the plant was unable to synthesise lignin.Explain the effect on the function of the leaf.Sewaktu pembentukan tisu xylem, satu tumbuhan gagal mensistesis lignin.

Terangkan kesannya keatas fungsi daun tumbuhan tersebut.

……………………………………………………………………………………………

……………………………………………………………………………………………

[2 marks]

(d) Based on diagram 1, state the meaning of cell specialization.Berdasarkan rajah 1, nyatakan maksud pengkhususan sel.

……………………………………………………………………………………………

……………………………………………………………………………………………

[2 marks]

(e) Leaf is the main photosynthetic organ of a plant.Explain the adaptation of tissue L to enable the leaf to carry out its function.Daun adalah organ utama fotosintesis sesuatu tumbuhanTerangkan adaptasi tisu L untuk membolehkannya menjalankan fungsinya

……………………………………………………………………………………………

……………………………………………………………………………………………………

……………………………………………………………………………………………………

……………………………………………………………………………………………………

[2 marks]

TOTAL

Page 22: BIO P1,P2 (2)

4551/2 - 4 - LONJAKAN SAUJANA SPM 2009

PPD MELAKA TENGAH

2. Diagram 2.1 shows the cell cycle of an organism.

Diagram 2.1

(a) Name phase U in Diagram 2.1..

U:………………………………………………………………………[1 mark]

(b) Phase U is further divided into three sub phases, X, Y and Z. Describe whathappens at sub phases X, Y and Z.

X: ………………………………………………………………………

Y: ………………………………………………………………………

Z: ………………………………………………………………………[3 marks]

(c) The number of chromosomes present in the nucleus of a somatic cell is 6.Diagram 2.2 shows a stage of cell division to produce gametes.

Diagram 2.2

Mitotic celldivision Phase U

Page 23: BIO P1,P2 (2)

4551/2 - 5 - LONJAKAN SAUJANA SPM 2009

PPD MELAKA TENGAH

Complete the diagram to show the chromosomes for a daughter cell producedat the end of sub-phase Q.

[2 marks]

( d ) Explain how radiotherapy affected cell cycle in cancer treatment.

…………………………………………………………………………………

…………………………………………………………………………………

…………………………………………………………………………………

[2 marks]

(e)(i) A farmer wants to breed a good variety of banana plants for commercialproduction. Suggest a suitable method to be used and explain how the methodnamed can increase the crop yield.

…………………………………………………………………………………

…………………………………………………………………………………

…………………………………………………………………………………

…………………………………………………………………………………

[3 marks]

(ii) State a problem that can occur when using this method..

…………………………………………………………………………………

…………………………………………………………………………………

[1 mark]

TOTAL

Page 24: BIO P1,P2 (2)

4551/2 - 6 - LONJAKAN SAUJANA SPM 2009

PPD MELAKA TENGAH

3. Diagram 3 shows part of a nitrogen cycle.Rajah 3 menunjukkan sebahagian kitar nitrogen.

DIAGRAM 4

DIAGRAM 3

a) Name P, Q and RNamakan P, Q dan R.

P: .........................................................................................................................

Q: ........................................................................................................................

R: .........................................................................................................................

[3marks]

Nitrogen in the atmosphereNitrogen dalam atmosfera

Nitrogen fixation bymicroorganisms in plant PPengikatan nitrogen olehmikroorganisma dalamtumbuhan P

Nitrogenouscompounds in plantsSebatian nitrogendalam tumbuhan

Nitrogenous compoundsin animalsSebatian nitrogen dalamhaiwan

Organism ROrganisma R

Process YProses Y

Substance QBahan Q

Ammonium compoundsSebatian ammonium

NitritesNitrit

Page 25: BIO P1,P2 (2)

4551/2 - 7 - LONJAKAN SAUJANA SPM 2009

PPD MELAKA TENGAH

(b) (i ) Name the microorganism that is involved in the nitrogen cycle and lives in plantP.Namakan mikroorganisma yang terlibat dalam kitar nitrogen dan tinggal dalamtumbuhan P.

..........................................................................................................................................[ 1 mark]

(ii) Besides nitrogen fixation by microorganisms, name a natural phenomenon which isalso able to convert nitrogen in the atmosphere to substance Q.Selain daripada pengikatan nitrogen oleh mikroorganisma, namakan satufenomena semulajadi yang boleh menukarkan nitrogen dalam atmosfera kepadabahan Q.

..........................................................................................................................................[ 1 mark]

(c) Microorganisms are involved in process Y.Mikroorganisma terlibat dalam proses Y.

(i) Name one type of microorganism which is involved in process Y.Namakan sejenis mikroorganisma yang terlibat dalam proses Y.

..........................................................................................................................................[ 1 mark]

(ii) Explain the role of the microorganism in ( c)(i)Terangkan peranan mikroorganisma dalam (c)(i)

..........................................................................................................................................

..........................................................................................................................................

..........................................................................................................................................

..........................................................................................................................................[ 3 marks]

(d) Explain how a deficiency of Q in the soil affects the growth of the plants.Terangkan bagaimana kekurangan Q dalam tanah memberi kesan terhadappertumbuhan tumbuhan.

..........................................................................................................................................

..........................................................................................................................................

..........................................................................................................................................[ 3 marks]

TOTAL

Page 26: BIO P1,P2 (2)

4551/2 - 8 - LONJAKAN SAUJANA SPM 2009

PPD MELAKA TENGAH

4 Two individuals P and Q were given injections to acquire immunity. The level of antibodiesin the blood of individual P and Q is shown in Diagram 4.1 and 4.2 respectively.

DIAGRAM 4.1

DIAGRAM 5.2

DIAGRAM 4.2

(a) What is the substance injected into the blood of individual P and individual Q ?

P : …………………………………………………………………………………

Q : …………………………………………………………………………………

1st injection 2nd

injection

0 1 2 3 4 5 6 7 8 9

Co

nce

ntr

ati

on

of

an

tib

od

ies

inth

eb

loo

d(a

rbit

rary

un

it)

Immunity level

Increaseimmediately

0 1 2 3 4 5 6 7 8 9

1st

vaccination 2nd

vaccination Time (weeks)

Immunity level

Co

nce

ntr

ati

on

of

an

tib

od

ies

inth

eb

loo

d(a

rbit

rary

un

it)

Booster dose (2nd)stimulates a faster and

larger lastingresponse.

Individual P

1st Injection 2nd Injection

Time (weeks)

Individual Q

Page 27: BIO P1,P2 (2)

4551/2 - 9 - LONJAKAN SAUJANA SPM 2009

PPD MELAKA TENGAH

[ 2 marks ]

(b) State the type of immunity obtained by individual P and individual Q.

P : …………………………………………………………………………….….

Q : ………………………………………………………………………………..

[ 2 marks ]

(c) Using your biological knowledge, describe how you could save this boy.

………………………………………………………………………………………………………

………………………………………………………………………………………………………

………………………………………………………………………………………………………

………………………………………………………………………………………………………

…………………………..…………………………………………………………………….……

[ 4 marks ]

(d) Table 4.1 shows a schedule of immunisation given for every new born Malaysian until the

age of two.

Age Types of Immunity

New born

Tuberculosis (B.C.G)

Hepatitis B ( First dose )

1 month Hepatitis B ( Second dose )

3 month

Triple Antigen

Polio ( First dose )

5 month

Triple Antigen

Polio ( Second dose )

Hepatitis B ( Third dose )

9 – 24 month Germans measles

1 ½ - 2 year

Triple Antigen

Polio ( Third dose )

TABLE 4.1

A boy was bitten by a snake. He was unconscious

and he was hospitalised.

Page 28: BIO P1,P2 (2)

4551/2 - 10 - LONJAKAN SAUJANA SPM2009

PPD MELAKA TENGAH

(d) (i) Based on Table 4.1, state the type of pathogen which cause the diseases.

...................................................................................................................................................

[1 mark]

(ii) Explain why there is a need for second and third doses for the immunisation.

………………………………………………………………………………………………………

………………………………………………………………………………………………………

………………………………………………………………………………………………………

………………………………………………………………………………………………………

[ 3 marks ]

TOTAL

Page 29: BIO P1,P2 (2)

4551/2 - 11 - LONJAKAN SAUJANA SPM 2009

PPD MELAKA TENGAH

5. Figure 5 shows various types of fingerprints.Rajah 5 menunjukkan bebrapa jenis cap jari

Composite Whorl Curves LoopsKomposit Pusar Lengkung Gelung

FIGURE 5

(a) (i) Based on Figure 5, name the type of fingerprints of students X and Y below.Berdasarkan Rajah 5, namakan jenis cap jari bagi pelajar X dan Y di bawah.

Student X Student Y

Type of fingerprint: Type of fingerprint:Jenis cap ibu jari: Jenis cap ibu jari:

……………………………… ….……………………………[2 marks]

(ii) State one factor that causes variation in the fingerprints of students X and Y.Nyatakan bagaimana faktor di (a)(ii) menghasilkan variasi.

……..………………………………………………………………………………….. ……..[1 mark]

(iii) State how the factor in (a) (ii) causes variation.Nyatakan bagaimana faktor di (a)(ii) menghasilkan variasi.

………………………………………………………………………………………………[1 mark]

Page 30: BIO P1,P2 (2)

4551/2 - 12 - LONJAKAN SAUJANA SPM 2009

PPD MELAKA TENGAH

(b) (i) What is the type of variation shown in Figure 5?Apakah jenis variasi yang ditunjukkan dalam Rajah 5?

…………………………………………………………………………………………………[1 mark]

(ii) State two traits, other than fingerprint, which show the same type of variation asin (b)(i).Nyatakan dua trait selain cap jari yang menunjukkan variasi yang sama seperti(b)(i).

Trait 1 : ……………………………………………………………………………………

Trait 2 : ……………………………………………………………………………………[2 marks]

(c) Height is a type of variation.Explain the differences between the type of variation shown by fingerprints andheight.

Trait ketinggian merupakan sejenis variasiHuraikan perbezaan antara variasi yang ditunjukkan oleh trait jenis cap jari dengantrait ketinggian pelajar.

…………………………………………………………………………………………………

…………………………………………………………………………………………………

[2 marks](d) Explain how variation can ensure the survival of a species.

Terangkan bagaimana variasi boleh menjamin kemandirian suatu spesies

…………………………………………………………………………………………………

…………………………………………………………………………………………………

…………………………………………………………………………………………………

[3 marks]

TOTAL

Page 31: BIO P1,P2 (2)

4551/2 - 13 - LONJAKAN SAUJANA SPM 2009

PPD MELAKA TENGAH

SECTION B

[ 40 marks ]

Answer two questions from this section.

6. Diagram 6.1 shows a respiratory structure of an insect.

Rajah 6.1 menunjukkan struktur respirasi satu serangga

DIAGRAM 6.1

(a) (i) Explain the gases exchange between tracheol and body cell.

Terangkan pertukaran gas antara trakeol dan sel-sel badan[4 marks]

(ii) Chitin is a polysaccharide on the outer surface of structure P. Due to the change inthe environment, the insect is unable to form the polysaccharide.Explain how the absence of chitin affects inhalation and the energy production.

Kitin adalah polisakarida yang terdapat pada permukaan struktur P. Disebabkanperubahan dalam persekitaran, serangga tidak dapat menghasilkan polisakarida.Terangkan bagaimana ketiadaan kitin memberi kesan keatas proses tarikan nafasdan penghasilan tenaga .

[6 marks]

PTracheolTrakeol

Body cellsSel-selBadan

Page 32: BIO P1,P2 (2)

4551/2 - 14 - LONJAKAN SAUJANA SPM 2009

PPD MELAKA TENGAH

(b) Diagram 6.2 shows the rate of oxygen intake before, during and after a vigorousexercise of an athlete.

Rajah 6.2 menunjukkan kadar pengambilan oksigen sebelum, semasa dan selepassatu latihan intensif seorang atlit.

Time (min)

DIAGRAM 6.2

(i) Based on the graph, compare the respiration before and during the vigorousexercise.

Berdasarkan graf diatas, bezakan proses respirasi sebelum dan semasalatihan tersebut.

[4 marks]

(ii) Explain how the oxygen intake by the athlete returns to the normal level at the 25thminute.

Terangkan bagaiman pengambilan oksigen oleh atlit tersebut kembali ke asalselepas minit ke 25

[6 marks]

Vigorous exercise

Oxygen intake(litre/minute)

Page 33: BIO P1,P2 (2)

4551/2 - 15 - LONJAKAN SAUJANA SPM 2009

PPD MELAKA TENGAH

(7) (a) Figure 7 shows development of the follicle in the female ovary, thickeningof uterine endometrium and the hormones involved.

Rajah 7 menunjukkan perkembangan folokel dalam ovari seorang perempuan,penebalan endometrium uterus dan hormonphormon yang terlbat.

FIGURE 7

Explain the relationship between development of the follicle , changing of therespective hormonal level in the blood and the thickening of the uterineendometrium in a female.

Terangkan hubungan antara perkembangan folikel, perubahan aras hormon-hormon masing-masing dalam darah dan penebalan endometrium uterus padaseorang perempuan.

(10marks)

Time (Day)

Estrogen

Page 34: BIO P1,P2 (2)

4551/2 - 16 - LONJAKAN SAUJANA SPM 2009

PPD MELAKA TENGAH

(b) Graph 7(a) and 7(b) show the growth curve of human and insect.Based on the graph , compare the growth process in human and insect.

Graf 7 (a) dan 7 (b) menunjkkan lengkuk pertumbuhan manusia dan seranggaBerdasarkan graf, bandingkan proses pertumbuhan pada manusia dan serangga.

(10marks)

GRAPH 7(a) : Growth curve for human

GRAPH 7(b) : Growth curve for insect

Height (cm)

Time (year)

Length(cm)

Time(day)

Page 35: BIO P1,P2 (2)

4551/2 - 17 - LONJAKAN SAUJANA SPM 2009

PPD MELAKA TENGAH

8 (a) Diagram 8 shows the blood groups of a married couple, Encik Ahmad andPuan Amalina and their children.

Rajah 8 menunjukkan kumpulan darah bagi pasangan suami isteri EncikAhmad dan Puan Amalina serta anak-anaknya.

ParentsIbu bapa

0ffspringAnak

blood group 0 blood group 0 blood group 0 blood group ABkumpulan darah O kumpulan darah O kumpulan darah O kumpulann darah AB

Diagram 8

Diagram 8 shows the variation of blood groups in En Ahmad’s family. Explain whythere is a variation in blood groups of the offspring.

Rajah 8 menunjukkan variasi kumpulan darah dalam keluarga En Ahmad. Terangkanmengapa adanya variasi dalam kumpulan darah anak-anaknya.

[ 10 marks]

(b) Genetic engineering is widely used in the field of agriculture and medicine.Justify the impact of genetic engineering on humans and the environment.

Kejuruteraan genetik digunakan secara meluas dalam bidang pertanian danperubatan.Beri wajaran tentang impak kejuruteraan genetik terhadap manusia danpersekitaran.

[ 10 marks]

E En AhmadBlood group AKumpulan darah AdarahBloogroup A

EPn AmalinaBlood group BKumpulan darah BBlood group A

Page 36: BIO P1,P2 (2)

4551/2 - 18 - LONJAKAN SAUJANA SPM 2009

PPD MELAKA TENGAH

Biodiversity is the variety of plants, animals and microorganisms living on Earth.These organisms live in different ecosystems and are important to our lives.Biodiversiti ialah kepelbagaian jenis tumbuhan, haiwan dan Mikroorganisma yanghidup di bumi. Organisma ini hidup dalam berbagai ekosistem dan penting kepadakehidupan kita.

9.(a)(i) Based on the statement discuss the importance of biodiversity.

Berdasarkan pernyataan di atas bincangkan kepentingan biodiversiti.[4 marks]

(ii) Diagram 9 shows an ecosystem in Malaysia.Rajah 9 menunjukkan satu ekosistem di Malaysia

Diagram 9

Discuss the importance of the ecosystem shown in Diagram 8 to the environmentand economy of our country.Bincangkan kepentingan ekosistem di Rajah 8 kepada persekitaran dan ekonominegara kita.

[6 marks](b)

Biotechnology is the application of organisms or microorganisms or their biologicalprocesses in the production of materials for use in medicine and industry.

Biotechnology ialah aplikasi organisma atau microorganism atau proses biologidalam penghasilan bahan-bahan untuk kegunaan bidang perubatan danperindustrian.

Discuss the uses of microorganisms inBincangkan pengunaan microorganisma dalam

(i) the waste treatment process.proses rawatan kumbahan.

(ii) food processingpemprosesan makanan

[10 marks]

END OF QUESTION PAPER

Page 37: BIO P1,P2 (2)

PPD MELAKA TENGAHLONJAKAN SAUJANA SPM 2009

JAWAPAN - BIOLOGY PAPER 1

1. A 11. C 21. C 31. C 41. C

2. B 12. C 22. C 32. D 42. A

3. A 13. A 23. C 33. D 43. A

4. A 14. D 24. C 34. C 44. B

5. B 15. D 25. D 35. C 45. B

6. D 16. C 26. B 36. C 46. C

7. C 17. B 27. C 37. C 47. D

8. B 18. D 28. D 38. C 48. B

9. A 19. A 29. B 39. B 49. B

10. A 20. C 30. C 40. D 50. D

Page 38: BIO P1,P2 (2)

PPDMT LONJAKAN SAUJANA SPM 2009 BIOLOGY P2

2

MARKING SCHEME - BIOLOGY PAPER 2

QUESTION 1

No Criteria Marks

(a) Able to name tissue K and tissue L.Answer: K: Upper epidermis (cells / tissue) L: Palisade mesophyll (cells / tissue)

11 2

(b) Able to state the function of cells K and M in a leaf.Sample answer: K: Protect the inner tissues. // Allows light to penetrate. M: Controls the size of stoma / transpiration / gaseous exchange

// Allows gaseous exchange through the stoma.

11

2

(c) (i) Able to explain the differentiation of cells J to form the xylem tissue.Sample answer: Cells J join end to end, / the wall of cells J at the joints dissolved, to form a hollow tube / continuous tube (from root to leaves). The wall of xylem vessel is thickened by lignin. (Any 2)

111 2

(ii) Able to explain the effect on the function of the leaf when the plantunable to synthesise lignin during the formation of the xylem tissue.Sample answer: Xylem cannot be strengthened / cannot uphold leaf. Less sunlight received / absorbed. Slow down the rate of photosynthesis / less glucose produced

Or (Any 2)

Xylem vessels collapsed. Less water supplied to leaves. Slow down the rate of photosynthesis / less glucose produced

(Any 2)

111

111 2

(d) Able to state the meaning of cell specialisation.Sample answer: Cells grow, change shape / differentiate. To carry out / perform specific function.

11 2

(e) Able to explain the adaptation of palisade mesophyll tissue to enablethe leaf to carry out its function.Sample answer: Upright and closely packed. Contains large number of chloroplast. All cells receive maximum amount of sunlight.

// Absorb maximum amount of sunlight // energy.

111

2

TOTAL 12

Page 39: BIO P1,P2 (2)

PPDMT LONJAKAN SAUJANA SPM 2009 BIOLOGY P2

3

QUESTION 2No Criteria Marks(a)

(b)

(c)

(d)

(e)(i)

(e)(ii)

Able to name the phase U.Sample answer:U : Interphase

Able to describe the processes at sub phases X, Y and Z duringphase U

Sample answer :

X : Cell synthesises protein / new orgenelles formedY : DNA is synthesized / is replicated / 2 sister chromatidsformedZ : Cell accumulates energy / synthesise energy / prepare forcell division

Able to draw a daughter cell based on the following criteria: No. of chromosomes are haploid / 3 chromosomes Types of chromosomes/ non homologous New genetic combination

Able to explain how radiotherapy can treat cancer.Sample answer :

F : Radiotherapy uses radiation / high energy raysE1 : destroy the nucleus of cancerous cellsE2 : cancerous cells die / cannot divide mitoticallyE3 : cell cycle stops

Able to name the method and explain the advantages of themethod in increasing crop yield.

Sample answer :T : Tissue culture / CloningE1 : Large numbers of clones can be producedE2 : Within a short period of time / any timeE3 : Clones inherited good characteristics/ resistance to diseases/ fast growth rate / large fruit / good genetic traits

Able to state one problem :Clones can be destroyed completely if they do not have theresistance to new diseases / pest.//No variation

1

11

1

111

Any 2

1111

Any1E = 1

1111

T=1mAny2E

Marks

Any 1

1

3

2

2

3

1

TOTAL 12

Page 40: BIO P1,P2 (2)

PPDMT LONJAKAN SAUJANA SPM 2009 BIOLOGY P2

4

QUESTION 3

No Criteria Marks

3 a

b(i)

(ii)

c(i)

(ii)

d

Able to name P,Q and RSuggested answerP: leguminous plant / example of a leguminous plantQ: nitratesR: denitrifying bacteria

Able to state the name of the microorganismSuggested answerRhizobium sp /nitrogen fixing bacteria

Able to name the natural phenomenon that can convert atmosphericnitrogen to substance QSuggested answerLightning

Able to suggest the type of the microorganism that is involved inprocess Y.Suggested answersaprophytic bacteria / fungi // putrefying bacteria / fungi

Able to explain the role of the microorganism in the nitrogen cycleSuggested answer

1. Saprophytic bacteria / fungi decompose protein in the deadplants and animals / excretory products of animals

2. to ammonium/ simpler nitrogenous compounds/ ammonia whichis eventually converted to nitrates.

3. This increases the nitrate / nitrogen content of the soil.

Able to explain how a deficiency of Q in the soil affects plant growth.Suggested answer1 Root hairs absorb less Q/nitrates/nitrogen// less Q/nitrates/nitrogen is available to be absorbed by plants.

2. Less chlorophyll / protein is synthesized.3 Rate of photosynthesis decreases.4 Plant growth is slow / retarded.

Any 3

111

1

1

1

1

1

1

1111

3

1

1

1

3

3

TOTAL 12

Page 41: BIO P1,P2 (2)

PPDMT LONJAKAN SAUJANA SPM 2009 BIOLOGY P2

5

QUESTION 4No Criteria Marks(a)

(b)

(c)

(d)(i)

(ii)

Able to state the substances injected into the blood of individualP and individual Q.Sample answer:P : Dead or weakened bacteria / viruses / antigens// vaccineQ : Serum containing antibodies // antiserum

Able to explain the type of immunity obtained by individual P andindividual Q.Sample answer :P : Artificial active immunity

The body produces its own antibodies to fight againstinfections by pathogens.

Q : Artificial passive immunityThe body receives antibodies produced from outside sourcesto fight against infections by pathogens.

Able to describe how could save that boy.Sample answer :F1: Snake venom / toxin acts as antigen to our bodyF2: Injection of serum which contains instant antibodies /antiserum / anti-toxin must be given to the patient.F3: Antibody-antigen action occured very fastF4: Antitoxin/ antibody reacts with toxin / snake venom/ antigenand neutralize it / he is saved.

Able to state the types of pathogen which cause the diseasesAnswer:Virus / bacteria

Able to explain why there is a need for second and third dosesfor the immunisationSample answer :

F1: Immunisation is given to prevent infection from pathogensthat caused diseases like Tuberculosis, Hepatitis B, Polio,diphtheria, whooping cough, tetanus. German measles ( state atleast 2 example )F2: New born are injected with vaccines to get Artificially ActiveImmunityF3: First dose are given to induce baby lymphocytes to produceantibodies which are specific against the antigens / bacteria /virusF4: 2nd and 3rd dose are booster dose to increase the productionof antibodies at a faster rate.F5: Achieved immunity level // antibodies remained in the bloodfor a long time and provide permanent immunity / protect themfrom the next infection.

Any 3

11

1

1

11

11

1

1

1

1

1

2

2

4

3

TOTAL 12

Page 42: BIO P1,P2 (2)

PPDMT LONJAKAN SAUJANA SPM 2009 BIOLOGY P2

6

QUESTION 5

No Criteria Marks

(a) (i)

( ii)

(iii)

(b) (i)

(ii)

(c)

(d)

Able to name the type of fingerprints of students X and YAnswer:X - Loop ; Y- Composite

Able to state one factor that causes variation in the fingerprintsof students X and Y.Answer:Genetic factor

Able to state how the factor in (a) (ii) causes variationAnswer:Genetic recombination during crossing overresults in the formation of different

Able to state the type of variationAnswer:Continuos variation

Able to state two traits, other than fingerprint, which show thesame type of variation as in (b)(i)Answer:

The ability to roll tongueTypes of hair

Able to explain the differences between the type of variationshown by fingerprints and height.Sample answer:

Height Types of fingerprint

- Shows normal distribution Shows discrete distribution

- Affected by environmental Not affected byFactors environmental factor

Able to explain how variation can ensure the survival of a species

Sample answer:

- Can differentiate from one individual to another / no one is thesame

- Able to adapt to a new environment- Able to camourflage to run away from any predators

2

1

1

1

11

1

1

1

1

1

2

1

1

1

2

2

3

TOTAL 12

Page 43: BIO P1,P2 (2)

PPDMT LONJAKAN SAUJANA SPM 2009 BIOLOGY P2

7

SECTION B

QUESTION 6

No Criteria Marks

(a) (i) Able to explain the exchange of gases between tracheole and bodycell.Sample answer: Partial pressure/concentration of oxygen in the tracheole is higher

than partial pressure/concentration of oxygen in body cell . Oxygen diffuse from tracheole to body cell Partial pressure/concentration of carbon dioxide in the body cell is

higher than partial pressure/concentration of carbon dioxide intracheole . Carbon dioxide diffuse from tracheole to body cell

1

1

1

1 4

(ii) Able to explain how the absent of chitin affect the process ofinhalation and energy production of the insect.

Sample answer: The function of chitin is to prevent trachea from collapsing/sustain

the air pressure During inhalation high pressure air moves into the trachea. The absent of chitin will cause the trachea / P to collapse / burst /

rupture. Air with oxygen cannot reach tracheal. Body cell cannot get enough oxygen for cellular respiration The insect does not produce enough energy and respire

anaerobically. Less energy produced. (Any 6)

1

11

111

16

b (i) Able to compare and explain the respiration before and duringvigorous exercise.Sample answer:

Before (A) During (B) Explanation (E)1.

AerobicRespiration

AnaerobicRespiration

Before - oxygen intake islow/the same as oxygenrequired/enough oxygenis supplied to the cell

During – oxygen requiredis more than oxygenintake

2.

Themuscles arein normalcondition

The musclesare in the stateof oxygen debt

Before – oxygen issufficient

During – oxygen isinsufficient / oxygensupplied is less thanoxygen supplied.

3.

Energyproduced is

Energyproduced is

Before – complete breakdown of glucose (produce

Page 44: BIO P1,P2 (2)

PPDMT LONJAKAN SAUJANA SPM 2009 BIOLOGY P2

8

more/38ATP

less / 2 ATP more energy) During – incomplete break

down of glucose (produceless energy)

4.

No/lessaccumulation of lacticacid in themuscles

Highaccumulationof lactic acid inthe muscles

Before – complete breakdown of glucose producecarbon dioxide and water

During – Incompletebreakdown of glucoseproduce lactic acid

A + B = 1m

E=1m (Any one E)

8

(b) (ii) Able to explain how the oxygen intake by the athlete returns to thenormal level at the 25th minute.Sample answer: Lactic acid has been removed from the muscles The lactic acid has been converted to energy/ convert to glucose

1

1 2

TOTAL 20

Page 45: BIO P1,P2 (2)

PPDMT LONJAKAN SAUJANA SPM 2009 BIOLOGY P2

9

QUESTION 7

No. Criteria Mark Remark7(a) Able to explain the relationship between development of

the follicle , changing of the respective hormonal level inthe blood and the thickening of the uterineendometrium.

Suggested answer:

Day 0 – 7Follicle– very small– start to develop when receive FSH from pituitary– the wall of follicle will produce estrogenFSH- pituitary start to release FSH, FSH will goto the ovary

- FSH stimulate development of follicle

Endometrium- stimulate by estrogen; undergo thickening / repairing

Day 8 – 14Follicle

- become larger, develop to form follicle GraafFSH / LH/ Estrogen/progesterone

- FSH decrease, LH at maximum level , estrogen atmaximum level

- LH stimulate ovulation / completion of meiosis I,estrogen stimulate the thickening of endometrium

- Progesterone level very low

Endometrium- endometrium become very thick (ready to

implantation (of embryo))-

Day 15 - 21Follicle

- Follicle undergoes ovulation/ released oocyte II- The remaining follicle tissue / corpus luteum

secreted small amount of estrogen but largeamount of progesterone

FSH/ LH/ Estrogen/ Progesterone- Progesterone stimulated the thickening of

endometrium , halted the secretion of FSH/LH- Development of new follicle and ovulation stop.

Endometrium- more thicker and highly vascular- ready for implantation of embryo

111

11

1

1

11

1

1

11

1

1

Max 3marks for

each stage- 9 marks

At least theanswer

shows therelationshipbetween 3parameteri.e follicle,hormone

andendometriu

m- 1 mark

Page 46: BIO P1,P2 (2)

PPDMT LONJAKAN SAUJANA SPM 2009 BIOLOGY P2

10

Day 22 – 28Corpus luteum

- if no fertilisation, corpus luteum becomedisintegrate

FSH/ LH/ Estrogen / Progesterone- FSH, LH and estrogen at minimum level;

progesterone level also drop

Endometrium- endometrium become breakdown & disintegrate- blood and tissue are shed / lining of uterus

discharge through vagina as menstrual flow.

1

1

1

1 Max – 10

7(b) Able to compare the growth process in human andinsect.Suggested answers:

Similarities- height of man / length of instar increases by time- both show horizontal line / constant growth during

adult

Difference- Form of graph – Sigmoid form for human and like

series of steps in insect- Age of organism – the height measured yearly,

but in insect used day for measuring the length- Caused of different – human have endoskeleton

but insect have exoskeleton- Stages involve – in human, the curve has three

different phases, but there are five steps in insect// nymphal stages

- Vertical and horizontal line : curve for human didnot shows different line (only the curve fromcontinuous points), but there are five differenthorizontal and vertical lines each

- Zero growth – no point to show zero growth inhuman, but there are 5 time of zero growth (athorizontal line)

- Sudden growth : no sudden growth for human,but there are sudden growth in insect (at verticalline)

- Ecdysis : no ecdysis in human but ecdysisoccurred in insect

- Mitosis : the cells in human undergo mitosis allthe time, but in insect, mitosis only occurred atcertain time (during ecdysis)

- Absorption of air : in human, there are noabsorption of air, but in insect, during ecdysis

1

1

1

1

1

1

1

1

1

1

1

1

Max – 10marks

*2 marksfor

similarities,8 marks fordifferences

TOTAL 20

Page 47: BIO P1,P2 (2)

PPDMT LONJAKAN SAUJANA SPM 2009 BIOLOGY P2

11

QUESTION 8

No Criteria Marks8(a) Able to explain why there is a variation of blood groups in the

offspring1. The ABO blood group in humans is controlled by three alleles

IA, , IB and Io.

2 Alleles IA and IB are codominant but allele Io is recessive.3 Ahmad is heterozygous for blood group A // Genotype of

Ahmad is IA, IO,

4 Amalina is heterozygous for blood group B // Genotype ofAmalina is IB, IO

5 Ahmad produces two types of sperms, one containing allele IA

and the other containing allele IO .6 Amalina produces ovum containing allele IO or allele IB

7 When the sperm containing allele IO fertilizes with the ovumcontaining allele IO the offspring produced will have thegenotype IO IO

8 and the phenotype is blood group O.9 Three of the children who have the blood group O are

produced this way and they have the genotype IOIO

10 When the sperm containing the IA allele fertilizes with theovum containing allele IB then the offspring produced willhave the genotype IA IB

11 and the phenotype is blood group AB.12 One of the children who have the blood group AB is produced

this way and has the genotype IA, IB.

Any 10 pointsGenetic diagram:

Parents Ahmad x AmalinaGenotype IA IO IB IO √PT 3,4

PT 5., 6Gametes√

Fertilisation

OffspringGenotype IAIB IOIO √PT7.10Phenotype blood group AB blood group B√ Pt 8, 11(If answer using schematic diagram - only maximum 7 marks)

1

11

1

1

11

11

1

1

1 10

No Criteria Marks

IAIO IB

IO

Page 48: BIO P1,P2 (2)

PPDMT LONJAKAN SAUJANA SPM 2009 BIOLOGY P2

12

8b Able to explain the effect of genetic engineering on man and theenvironmentSuggested answerGood effectsAgricultureF1 Genetic engineering used to produce disease resistant/ pest resistantplants

e.g legumes, peas and beansP : Less pesticides are used

- less pollution to the environment- better health for consumers.

P : increase yield of crops- better livelihood for farmers.- help to solve problems of insufficient food.

F2 : create crops with better nutrition value e.g tomatoes with highervitamin A content

- help to solve problems of malnutrition.F3: create crops with longer shelf lives e.g tomato

- less food wastageF4 : genetically modified livestock e. g cows

- produce meat with less fat / more milk.

MedicineF5 : genetically modified bacteria produce insulinP ; for treatment of diabetis mellitusF6: Genetically modified yeast to produce vaccine for hepatitisP: for prevention of diseases.F7: Gene therapy for treatment of genetic disorders/ diseases e.gmuscular dystrophy, rheumatoid arthritis, sickle cell anaemiaP: Defective gene removed and normal gene inserted.

Any 2F and P for agriculture,any 1F and P for medicine Max : 6m

Bad effectF1 Pest resistant genes may be transferred to weedsP: may be difficult to control growth of weeds.F2: Some transgenic crops may have animal genesP : this may not be acceptable to certain groups for religious reasons.F3: Genetically modified foods may be harmful to healthP: may activate human genes to cause cancer.F4: Transgenic organisms may affect the survival of other organisms

in the ecosystem.P: may cause the imbalance of nature / ecosystemF5: Gene therapy used for the treatment of genetic disorder has itslimitations.P : may not be acceptable because of religious and moral values.

: very costlyAny 2F and P

Max 4m

6

410

TOTAL 20QUESTION 9

Page 49: BIO P1,P2 (2)

PPDMT LONJAKAN SAUJANA SPM 2009 BIOLOGY P2

13

No Criteria Marks

9 (a)(i)

(ii)

Able to discuss the importance of Biodiversity

Suggested answer:

F- it provides humans with necessities of lifeP- is a resource for food/timber to build shelter/as a fuel/fibresfor clothing.F- many plant species are original sources of pharmaceuticaldrugs/medicines.P- new commodities, for example, new crop plants or medicinalmaterials could be developed using the gene pool from wildspecies in the forests.F- allow for biological control to maintain stable populationP- regulate climatic conditions, biogeochemical cycles, preventfloodingF- Natural ecosystems and species in the wild are beautifulP- there is much pleasure to be derived from unspoilt naturalenvironmentF- ecotourismP- could provide income for some countries

Any 4 points

Able to discuss the Importance of mangrove swamps.Sample answer:

- F: resource for timber used in building industry- P : has many varieties of species of mangrove trees- F: Mangroves protect the coastlines and prevent- coastal erosion.- P: The roots of mangrove trees act as wave- breakers which stabilise the coastlines.- F:: Mangrove swamps are good breeding grounds- for fish and crustaceans/ prawns, crabs etc.- Important for fishermen- P: The calm water and prop roots shelter the- spawns from predators- F: Serve as habitat for many species of birds,- amphibians and reptiles.- P: The habitat provides food, shelter, living space,- nesting and breeding sites for these animals.- F: Serve as natural barriers against torrential- storms and tsunamis- P: The trees block the water from flooding the

land during a storm.Any 6 ponts

Max 4

Max 6

Page 50: BIO P1,P2 (2)

PPDMT LONJAKAN SAUJANA SPM 2009 BIOLOGY P2

14

(b)(i)

(ii)

Able to discuss the uses of microorganisms in ;

Waste treatment- rich in organic matters, bacteria and microorganisms- (in oxidation pond)the sewage is decomposed

by(millions) of aerobic bacteria(in the presence ofoxygen)

- Decomposed sewage/sludge settled to the bottom of thepond

- fermentation takes place at sedimentation tanks

- using anaerobic bacteria- produce methane/carbon dioxide/minerals- digested sludge use as fertilizers

Any 5 points

Food processing- F: Use of yeast in making of bread and cake- P: Fermentation by yeast produces carbon dioxide- which makes dough rise.- F: Beer brewed from barley/ wine from grape juice- P: Yeast fermentation of the sugar in barley/ grape- produces ethanol- F: Yoghurt is made from fermentation of milk by- bacteria / Lactobacillus sp. / Streptooccus- thermophillus- P: Bacteria converts sugar into lactic acid which- coagulates the milk to / form yoghurt- F: Cheese made by adding bacteria and rennin to- milk.- P: Milk separates into curd and whey/ coagulates- F: Soya sauce made from fermentation of soya- bean by fungi- P: Yeast fermentation breaks down soya bean and- gives it flavour

Any 5 points

1

1

1

1

111

Max 5

Max 5

TOTAL 20

Page 51: BIO P1,P2 (2)

j*k

Page 52: BIO P1,P2 (2)

j*k

Page 53: BIO P1,P2 (2)

j*k

Page 54: BIO P1,P2 (2)

j*k

Page 55: BIO P1,P2 (2)

j*k

Page 56: BIO P1,P2 (2)

j*k

Page 57: BIO P1,P2 (2)

j*k

Page 58: BIO P1,P2 (2)

j*k

Page 59: BIO P1,P2 (2)

j*k

Page 60: BIO P1,P2 (2)

j*k

Page 61: BIO P1,P2 (2)

j*k

Page 62: BIO P1,P2 (2)

j*k

Page 63: BIO P1,P2 (2)

j*k

Page 64: BIO P1,P2 (2)

j*k

Page 65: BIO P1,P2 (2)

j*k

Page 66: BIO P1,P2 (2)

j*k

Page 67: BIO P1,P2 (2)

j*k

Page 68: BIO P1,P2 (2)

j*k

Page 69: BIO P1,P2 (2)

j*k

Page 70: BIO P1,P2 (2)

j*k

Page 71: BIO P1,P2 (2)

j*k

Page 72: BIO P1,P2 (2)

j*k

Page 73: BIO P1,P2 (2)

j*k

Page 74: BIO P1,P2 (2)

j*k

Page 75: BIO P1,P2 (2)

j*k

Page 76: BIO P1,P2 (2)

j*k

Page 77: BIO P1,P2 (2)

j*k

Page 78: BIO P1,P2 (2)

j*k

Page 79: BIO P1,P2 (2)

j*k

Page 80: BIO P1,P2 (2)

j*k

Page 81: BIO P1,P2 (2)

j*k

Page 82: BIO P1,P2 (2)

j*k

Page 83: BIO P1,P2 (2)

j*k

Page 84: BIO P1,P2 (2)

j*k

Page 85: BIO P1,P2 (2)

j*k

Page 86: BIO P1,P2 (2)

j*k

Page 87: BIO P1,P2 (2)

j*k

Page 88: BIO P1,P2 (2)

j*k

Page 89: BIO P1,P2 (2)

j*k

Page 90: BIO P1,P2 (2)

j*k

Page 91: BIO P1,P2 (2)

j*k

Page 92: BIO P1,P2 (2)

j*k

Page 93: BIO P1,P2 (2)

j*k

Page 94: BIO P1,P2 (2)

j*k

Page 95: BIO P1,P2 (2)

j*k

Page 96: BIO P1,P2 (2)

j*k

Page 97: BIO P1,P2 (2)

j*k

Page 98: BIO P1,P2 (2)

j*k

Page 99: BIO P1,P2 (2)

j*k

Page 100: BIO P1,P2 (2)

j*k

Page 101: BIO P1,P2 (2)

j*k

Page 102: BIO P1,P2 (2)

j*k

Page 103: BIO P1,P2 (2)

MARKING SCHEME PAPER 1 TRIAL KEDAH 2010 1. C 26. A 2. A 27. A 3. C 28. B 4. D 29. C 5. C 30. C 6. B 31. C 7. A 32. B 8. D 33. A 9. D 34. C

10. B 35. D 11. C 36. A 12. D 37. A 13. D 38. A 14. D 39. D 15. B 40. A 16. D 41. A 17. C 42. C 18. A 43. B 19. D 44. B 20. C 45. A 21. B 46. B 22. D 47 D 23. A 48. A 24. B 49. C 25. C 50. B

j*k

Page 104: BIO P1,P2 (2)

2

BIOLOGY SECTION A

PAPER 2 [4551/2] No.

Marking Criteria / Sample Answers Marks

1 (a) (i) Gills

1

(ii) Tracheal system

1

(b) P : Filaments Q: Spiracles

1 1

2

(c) (R is ring of chitin which) support the tracheal / prevent the tracheal from collapsing.

1

(d) Diagram 1.1(b): P1: The filament have numerous thin-walled lamellae to maximise the surface area for gaseous exchange. P2: The gill filaments have thin membrane and covered by a net work of capillaries to transport respiratory gases. P3: The surface of the gills is moist which allows the gases to be dissolved. Any 1P

1

Diagram 1.2(b) P1: The large number of tracheoles provides a large surface for the diffusion of gases. P2: Tip of tracheoles have thin permeable walls and contain fluid in which respiratory gases can dissolve. P3:Terminal ends of the tracheol remains moist which allows the gases to be dissolved. Any 1P

1

(e) (i) P1:( The gaseous exchange process occurs over the whole body surface in an Amoeba sp) through simple diffusion. P2:Higher concentration of oxygen in the water surrounding causes oxygen to diffuse into the Amoeba. P3:Higher concentration of carbon dioxide in the cell causes carbon dioxide to diffuse out of the Amoeba. Any 2P

1 1 1

2

(ii) S: Contractile vacuole

1

(iii) P1: Freshwater is hypotonic to the cytoplasmic fluid of Amoeba sp . P2: Water diffuses into the cell and fill the contractile vacuole by osmosis P3: When the contractile vacuole is filled with water to its maximum size, it contracts to expel its content from time to time. Any 2P

1 1 1

2

j*k

Page 105: BIO P1,P2 (2)

3

No.

Marking criteria/ Sample answers Mark

2 (a) (i) Osmosis

1

(ii) P1 : Sucrose solution is hypertonic / more concentrated. P2 : Water diffuse from distilled water into the sucrose solution P3 : The level of sucrose solution in the capillary tube stop rising at the equilibrium stage / the concentration inside and outside of the visking tubing is the same / the amount of water diffuse into and out from the visking tubing is the same. Any 2 Ps

1 1 1

2

(b)

F- Sucrose molecules are too large E- The visking tubing is a semi permeable membrane/ which only allows certain substances to pass through.

1 1

2

(c) (i) Y : crenation Z : haemolysis

1 1

2

(ii) P1- Solution Z is hypotonic compare to red blood cell. P2- Osmosis occur P3- water leaves/ diffuses into the cell P4- Red blood cell expand/ swell and burst. Any 3P

1 1 1 1

3

(iii) F : No P1 : Plant cell consists of cell wall P2 : Cell wall is made up of cellulose // Cell wall able to withstand the pressure. Any 2

1 1 1

2

Total

12

j*k

Page 106: BIO P1,P2 (2)

4

No.

Marking criteria/ Sample answers Mark

3 (a) (i) Absorption / Simple diffusion / facilitated diffusion

1

(ii) F1 thin wall/ one cell thick E1 increase rate of diffusion of digested food/ nutrients F2 large surface area/ has microvilli E2 increase rate of absorption of digested food/ nutrient F3 has a network of capillaries/ blood vessels E3 to transport the absorbed nutrients Any F + E

1 1 1 1 1 1

2

(b)

P: hepatic portal vein Q: lymphatic/lymph vessel/ duct

1 1

2

(c) P1: Deamination.// The amino group is removed (from amino acid)/ converted to ammonia . P2: (Ammonia) is converted to urea. P3: urea will be excreted through the kidneys. Any 2 Ps

1 1 1

2

(d) L1: A major energy reserve in the body// L2: (phospholipids are) components of the plasma membrane// L3: Lipids is used as a respiratory substrate// L4: Excess fats are stored in adipose tissues (under the skin, around internal organs) Any 1L A1:Amino acids are used in protein synthesis// A2:For repair and production of new protoplasm/growth and repair// A3:Used in the formation of enzymes/ some hormones/protein part of haemoglobin/ antibodies Any 1A G1:Glucose is used as the main respiratory substrate// It is oxidised to release energy (water and carbon dioxide)// G2:Excessive glucose is converted to glycogen // Blood glucose level rise / increase. Any 1 G

1 1 1 1 1 1 1 1 1

3

(e) P1: Diabetes mellitus // Blood sugar level increases// Hyperglycemia P2: Excess glucose cannot be converted to glycogen.

1 1

2

Total 12

j*k

Page 107: BIO P1,P2 (2)

5

No.

Marking criteria/ Sample answers Mark

4 (a)

Both arrows correct

1

(b) A – Pulmonary artery B – Pulmonary vein

1 1

2

(c)

F : Contraction of ventricle / heart E1: generates a (high) pressure E2 : (to) propel/ force / pump the blood flow from the heart/ ventricle to vessel A Any two

1 1 1

2

(d)(i)

Coronary artery

1

1

(ii) P1: Cut the supply of O2/ nutrients to the heart muscle P2: causing chest pain / angina / heart attack / myocardial infarction Reject ‘Heart problem’

1 1

2

(e) (i)

(ii)

P1: platelets break down and release chemicals P2: to cause platelets to stick to each other P3: platelets clump together to form a plug to prevent blood loss . P4: released thrombokinase and other clotting factors Any 2P P1 : Fibrinogen is soluble, fibrin is insoluble / not soluble P2 : Fibrin able to form fibres / meshwork / thread to trap blood cells, fibrinogen is not able to do so.

1 1 1 1 1 1

2 2

Total 12

j*k

Page 108: BIO P1,P2 (2)

6

No.

Marking Criteria / Sample Answers Marks

5 (a) (i) (Transfer/flow of) energy

1

(a) (ii) F : Phytoplankton is an autotrophic organism. P1 : Able to absorb light energy / consists of chloroplast. P2 : synthesis their own food / carry out photosynthesis Any 2

1 1 1

2

(b) F1 : population of small fish increases P1 : no shark feed on small fish // shark is the predator F2 : population of plankton decreases P2 : more small fish feed on the plankton F3 : Eventually the population of small fish decreases Any 3

1 1 1 1 1

3

(c) F : Commensalism P1 : Shark is the host / neither gain any benefit nor harmed. P2 : Remora benefits P3 : Remora obtain protection / food / transport from the shark. Any 3

1 1 1 1

3

(d) P1 : Fertilizer washed away by rain water into the lake P2 : Nutrient / minerals content in the lake increase. P3 : alga bloom / alga grow rapidly in the lake. P4 : eutrophication occur. P5 : Oxygen content in the lake decrease / drop P6 : Fishes die / population decrease Any 3 P

1 1 1 1 1 1

3

Total

12

j*k

Page 109: BIO P1,P2 (2)

7

BIOLOGY SECTION A

PAPER 2 [4551/2] - ESSAY

No.

Marking Scheme Mark

6(a) (i)

Continuous variation : body weight, height Discontinuous variation : types of earlobe, types of finger print.

1 1

2

(a)(ii) Continuous Variation Discontinuous variation P1 The changes of

characteristics among individual are gradual

The differences among individuals are distinct.

P2 Continuous variation is quantitative // characteristics can be measured.

Discontinuous variation is qualitative // characteristic is either present or absent.

P3 The graph shows the normal distribution curve.

The graph shows the discrete distribution.

P4 The character is determined by many genes

The character is determined by a single genes

P5 The characteristic is influenced by the environmental factor and genetic factor.

The characteristic is influenced by the genetic factor.

P6 Exhibits a range of phenotype with intermediate characters.

There are no intermediate groups.

Any 4 pair

Max 4 m

(b) Albinisme F : Albinisme is caused by the change in gene // mutation P1 : Body / skin unable to produce black pigment (melanin) P2 : The skin and hair of albinos are white // their eyes are pink. Any 2 Sickle cell anaemia F : Sickle cell anaemia is caused by the change in the genes // mutation. P1 : haemoglobin produced is not normal / abnormal P2 : Abnormal haemoglobin unable to bind / transport / carries with oxygen efficiently. P3 : The patient will always feel weak / cannot carries out vigorous activities. Any 2

1 1 1 1 1 1 1

Max 2 m

Max 2m

j*k

Page 110: BIO P1,P2 (2)

8

6(c) (i) Abiotic factors that cause variation between the two sets of ginger plants are: F1: Sun light P1: Plants need light energy to carry out photosynthesis for growth P2: Set A, plants are obtain more / exposed to sunlight // Plants in set B obtain less sunlight / not exposed to Sunlight. P3 : Growth rate of plants in Set A is higher than plants in Set B.

1 1 1 1

F2: Space P4: Plants need (space) to grow a large root system / leaves P5: Plants able to absorb sufficient water and minerals/sunlight. P6: Set A, plants have larger space for the root and leaves to Grow // Plants in set B have smaller space for the root and leaves to grow.

1 1 1 1

F3: Soil / minerals P7: Plants need mineral for (healthy) growth. P8: Loam soil provides more minerals in Set A. // Sandy loam soil in Set B contains less minerals. P9: Loam soil able to trap / store water better than sandy loam soil. Any 8

1 1 1 1

max 8

6(c) (ii) F1 : Plantlets from tissue culture have the same genetic material. P1 : This is to show /ensure/proof the differences of plants in Set A and Set B are not caused by genetic factor / have the same genetic material. // This is to show /ensure/proof the differences of plants in Set A and Set B are caused by abiotic factors.

1 1

2

Total 20 No.

Marking Scheme Mark

7(a) P1 : Nerve impulses arrive at the axon terminal of (presynaptic) neurone. P2 : Causes the synaptic vesicles to move towards the (presynaptic) membrane and fuse with the membrane. P3 : Neurotransmiters /acetylcoline (examples) molecules are released from synaptic vesicles. P4 : (The neurotransmitter molecules) diffuse across the synaptic cleft into the postsynaptic knob / dendrite / cell body of neighbouring neurone.. P5 : The neurotransmitter molecules bind to specific receptor sites in the postsynaptic knob. P6 : The binding triggers / generates new nerve Impulses. P7 : The impulses then move along the postsynaptic neurone. P8 : The release of neurotransmitter is in one direction, from the synaptic knob to the postsynaptic neurone. P9 : Mitochondria in the synaptic knob generate ATP / energy to synthesis neurotransmitter molecules. Any 6

1 1 1 1 1 1 1 1 1

Max 6

j*k

Page 111: BIO P1,P2 (2)

9

No.

Marking Scheme Mark

7 (b) P1 : The receptor at the terminal of X stimulated by the heat. P2 : The receptor generates a nerve impulse. P3 : The nerve impulse travels along X / afferent neurone To the spinal cord. P4 : In the spinal cord, the nerve impulse is transmitted to an interneurone. P5 : From the interneurone, the nerve impulse is transmitted to an efferent neurone/ neurone Y. P6 : Nerve impulse travels along efferent neurone / Y and reach the effector / muscle tissue / fingers. P7 : Muscles contract to withdraw the hand / finger. Any 4

1 1 1 1 1 1 1

Max 4 7 (c) P1 : The receptors in the eyes detect the dog.

P2 : Nerve impulses are generated and transmitted to the brain via the afferent neurone. P3 : The hypothalamus in the brain is stimulated. P4 : It actives the sympathetic nervous system to generate nerve impulses. P5 : Nerve impulses are transmitted to the adrenal medulla to stimulate secretion of adrenaline. P6 : Adrenaline carried / transported by blood circulatory system to the targeted organs. P7 : Adrenaline promotes the breakdown of glycogen to glucose. P8 : (Adrenaline) increases the breathing rate. P9 : More oxygen will be taken into the body P10 : (Adrenaline) increases the rate of heartbeat/ blood pressure. P11 : Rate of the blood flow increase. P12 : More glucose and oxygen will be supplied to the muscles. P13 : More energy produced by the muscles. // metabolic rate increase. P14 : Body has enough energy to face the ‘fight or flight’ situation. Any 10

1 1 1 1 1 1 1 1 1 1 1 1 1 1

Max 10

Total 20

j*k

Page 112: BIO P1,P2 (2)

10

No. Marking Scheme Mark 8 (a)(i) P1 fish have streamline shapes // the anterior of the fish is

smooth and rounded // the body is long and tapers towards the end.

P2 the body of a fish is covered with scales that have a slimy coating

1 1

2

(a)(ii) P1 myotomes muscles are arranged in both side of the body P2 the vertebral column of the fish is flexible and can bent

from side to side P3 myotome muscles act antagonistically in fish./ carry out

opposite action in a fish P4 when the muscles on right side contract, the muscle on

the left side relax P5 the tail/body will be bent to the right. P6 when the muscles on left side contract, the muscle on

the right side relax P7 the tail/body will be bent to the left. P8 alternate contraction of the right and left myotome block

enable its tail to move left and right P9 to produce a force that propel the fish forward.

[ any 6]

1 1 1 1 1 1 1 1 1

Max 6

(b)(i) Similarities: F1 Both Joint S and Joint T has a cavity filled with svnovial fluid // lined with synovial membrane El Synovial fluid acts as lubricant to reduce friction between bones // absorbs shock of the movement. F2 The end surfaces of the humerus bone of Joint S and Joint T are covered with cartilage E2 To protect the bone / reduce friction between the bones F3 Both Joint S and T are connected with ligaments E3 to absorb shock // strengthen the articulation of bones/ joint. Differences: D1 Joint S is hinge joint E4 Joint S allows the movement of bones in one plane / direction D2 Joint T is ball-and-socket joint. E5 Joint T allows rotational movement of bones in all directions.

[ any 8 ]

1 1 1 1 1 1 1 1 1 1

Max 8

8 (b)(ii) Osteoporosis P1 : the bone become thinner / more brittle / porous / fragile. P2 : Loss of bone mass. P3 : Lack of calcium / phosphorus / vitamin D Arthritis P4 : Cartilage between bones become thinner. P5 : Ligaments become shorter / loss elasticity P6 : Less production of synovial fluid. P7 : The joints become swollen / stiff / painful

[ any 4 ]

1 1 1 1 1 1 1

Max 4

Total 20

j*k

Page 113: BIO P1,P2 (2)

11

No.

Marking Scheme Mark

9 (a)

(b)

The tree F1 : Less tree will be chopped / felled P1 : More CO2 absorbed by the trees for photosynthesis P2 : Avoid the increasing of CO2 in the atmosphere. P3 : Reduce the impact of Green house effect // global warming P4 : Less habitat of fauna and flora will be destroyed. P5 : Reduce / avoid the extinction of fauna and flora. P6 : To maintain / preserve the biodiversity. The oil / fuel // Save Energy F2 : Reduce the burning of oil / fuel P7 : More fuel/energy can be preserved for future. P8 : Less green house gases / acidic gases released. P9 : Reduce / avoid the impact of green house effect / acid rain. The Landfill F3 : Less landfill will be opened P10 : Landfill cause leaching / ground water pollution. P11 : Less diseases / health problem caused by the improper managed landfill. The Water F4 : Less used water / effluent / untreated sewage released into river. P12 : Reduce / avoid the impact of water pollution / avoid the extinction of aquatic organisms. Any 10 Good Effect G1 : Generate hydropower electricity G2 : As reservoir / to store water / supply fresh water G3 : Supply water for agricultural / industries. G4 : Place/site for recreation / tourism G5 : Reduce the flood problem at the downstream. Bad Effect B1 : Flooded / submerge trees / habitat of the fauna and flora B2 : Less tree / plants to carry out photosynthesis // Less CO2 absorbed for photosynthesis B3 : Amount of CO2 in the atmosphere increase B4 : Increase the impact of green house effect / global warming. B5 : Many species of fauna and flora extinct // Reduce the biodiversity. B6 : Reduce the flow of water at the downstream. B7 : Cause the population of aquatic life at the downstream reduce. B8 : Reduce the land used for residential / agricultural B9 : Flooded / destroy / loss of historical building / site. Any 10

1 1 1 1 1 1 1 1 1 1 1 1 1 1 1 1

1 1 1 1 1 1 1 1 1 1 1 1 1 1

max 10

max 10

Total 20

j*k

Page 114: BIO P1,P2 (2)

SULIT 4551/1

1. Which organ consists of cells which has the highest density of rough endoplasmic reticulum ?

Organ manakah mengandungi sel-sel yang mempunyai kepadatan jalinan endoplasmik kasar paling tinggi?

A Stomach C Brain Perut Otak B Heart D Kidney Jantung Ginjal

2. Diagram 1 shows three types of cells. Rajah 1 menunjukkan tiga jenis sel.

P Q R Diagram 1

To which systems do the cells shown above belong? Kepada system manakah sel-sel di atas dipadankan ?

P Q RA. Digestive system

Sistem pencernaan]Respiratory system

Sistem respirasiNervous system

Sistem sarafB. Nervous system

Sistem sarafReproductive systemSistem pembiakan

Circulatory systemSistem peredaran

C.

Respiratory systemSistem respirasi

Circulatory systemSistem saraf

Digestive system Sistem pencernaan

D.

Reproductive systemSistem pembiakan

Digestive systemSistem pencernaan

Nervous systemSistem saraf

[Turn over CONFIDENTIAL

Page 115: BIO P1,P2 (2)

SULIT 4551/1

3. Diagram 2 shows a unicellular organism living in freshwater pond.Rajah 2 menunjukkan sejenis organisma unisel yang hidup di dalam kolam air tawar.

Which process involves in the movement of water ?Apakah proses yang terlibat dalam pergerakan air.

A Diffusion C Active transportResapan Pengangkutan aktif

B Osmosis D Facilitated diffusionOsmosis Resapan berbantu

4. Diagram 3 shows a cross section of a leaf.Rajah 3 menunjukkan keratan rentas daun

Diagram 3

Which of the cell labelled A, B, C and D does not contain chloroplast?

Antara sel yang berlabel A, B, C dan D yang manakah tidak mengandungi kloroplas?

[Turn over CONFIDENTIAL

Diagram 2

WaterAir

A

B

C

D

Page 116: BIO P1,P2 (2)

SULIT 4551/1

5. Diagram 4 shows a type of plant tissue. Rajah 4 menunjukkan sejenis tisu tumbuhan.

Diagram 4

What is the importance of the thickening of substance X to the plant tissue? Apakah kepentingan penebalan bahan X terhadap tisu tersebut ?

A To transfer photosynthesis products Untuk memindahkan hasil fotosintesis

B To give turgidity to the tissues Untuk memberikan kesegahan kepada tisu

C To transfer water and mineral salts Untuk memindahkan air dan garam mineral

D To give support and mechanical strength

Untuk memberikan sokongan dan kekuatan mekanikal

6. Which of the following sequence of organelles involved in the synthesis of extracellular enzymes is correct? Manakah di antara urutan berikut betul yang melibatkan organel dalam sintesis enzim luar sel? A Golgi apparatus→Ribosomes →Rough endoplasmic reticulum Alat Golgi - Ribosom Jalinan endoplasmik kasar B Rough endoplasmic reticulum→Ribosomes→Golgi apparatus Jalinan endoplasmik kasar Ribosom Alat Golgi C Ribosomes→Golgi apparatus→Rough endoplasmic reticulum Ribosom Alat Golgi Jalinan endoplasmik kasar D Ribosomes→Rough endoplasmic reticulum→Golgi apparatus Ribosom Jalinan endoplasmik kasar Alat Golgi]

[Turn over CONFIDENTIAL

Page 117: BIO P1,P2 (2)

SULIT 4551/1

7. Carrot slices are immersed in 0.1% sucrose solution. After 4 hours, the slices are found to be turgid and hard. Hirisan lobak merah direndam di dalam larutan sukrosa 0.1%. Selepas 4 jam, hirisan itu didapati segah dan keras.

Which of the following statement explains this phenomenon? Antara pernyataan berikut, yang manakah menerangkan fenomena ini ?

A The carrot cell wall prevent it from shrinking . Dinding sel karot menghalangnya dari mengecut. B The high concentration of the cell sap in the vacuole causes water to diffuse. . Kepekatan yang tinggi dalam sap sel vakuol menyebabkan air meresap ke dalam Sel. C The cell sap is hypotonic towards the sucrose solution. Sap sel adalah hipotonik kepada larutan sukrosa. D The carrot cell wall allows the sucrose molecules to diffuse into the cell. Dinding sel karot membenarkan molekul selulosa meresap ke dalam sel. 8. Diagram 5 shows a cell after immersed into a particular solution. Rajah 5 menunjukkan sel yang telah direndamkan ke dalam larutan tertentu.

Diagram 5

Which is experienced by the cell? Apakah yang dialami oleh sel itu ?

A Crenation C Deplasmolysis Krenasi Deplasmolisis B Plasmolysis D Haemolysis Plasmolisis Hemolisis

[Turn over CONFIDENTIAL

Page 118: BIO P1,P2 (2)

SULIT 4551/1

9. Diagram 6 shows the action of an enzyme on a substrate. Rajah 6 menunjukkan tindakan enzim ke atas suatu substrat.

What is represented by K?Apakah yang diwakili oleh K?

A Enzyme C Products of reaction Enzim Hasil tindak balas

B Enzyme-substrate complex D Substrate Kompleks-enzim substrat Substrat

10. Based on the information below, name the enzyme that can be used.Berdasarkan maklumat di bawah, namakan enzim yang sesuai digunakan.

Extracting agar jelly from seaweeds Mengasingkan agar-agar daripada laut.]

Removing the seed coats from cereal grains Mengeluarkan kulit dari bijirin

A Zymase C Celulase Zimase Selulosa

B Amylase D Protease Amilase Protease

11. Diagram 7 shows a graph between the rate of reaction at different substrate concentration when factor Q is changed. Rajah 7 menunjukkan graf diantara kadar tindak balas dan kepekatan substrat apabila faktor Q diubah.

[Turn over CONFIDENTIAL

Diagram 6

K

Page 119: BIO P1,P2 (2)

SULIT 4551/1

Diagram 7

What is factor Q ? Apakah faktor Q ?

A pH C Inhibitor pH Perencat B Time D Enzyme concentration Masa Kepekatan enzim

12. The following information shows starch molecules undergoing process M. Maklumat berikut menunjukkan molekul kanji melalui proses M.

Process M Process M Starch ------------------ Maltose -------------------- Glucose [Kanj] Maltosa Glukosa

What is process M ? Apakah proses M ?

A Photosynthesis C Condensation Fotosintesis Kondensas] B Hydrolisis D Polymerisation Hidrolisis Pempolimeran

[Turn over CONFIDENTIAL

Factor QFaktor Q

Page 120: BIO P1,P2 (2)

SULIT 4551/1

13. Diagram 8 shows a cell cycle of an organism. Rajah 8 menunjukkan kitar sel bagi suatu organisma.

Diagram 8 Which of the following represent X and Y ? Yang manakah di antara berikut mewakili X dan Y ?

Phase X Phase YA Meiosis InterphaseB Interphase MitosisC Mitosis InterphaseD Interphase Meiosis

14. The diploid chromosomes in a leaf cell of a maize plant is 20. If one of the homologous chromosome pair does not separate during the Meiosis 1, how many chromosomes can be found in the male nucleus of a pollen grain of maize ?

Nombor kromosom diploid dalam daun jagung ialah 20. Jika satu daripada Pasangan kromosom homolog tidak terpisah semasa Meiosis 1, berapakah bilangan kromosom yang mungkin didapati pada debunga daun jagung ?

A 9 B 10 C 20 D 18

15. Which of the following statements explain the importance of mitosis to cells? Yang manakah di antara pernyataan berikut menerangkan kepentingan mitosis kepada sel ?I . To ensure the chromosomal number is constant in all somatic cells. Untuk memastikan bilangan kromosom adalah tetap dalam semua sel somatik. II To ensure the daughter cells have the same number of chromosomes as the parent cell. Untuk memastikan billangan kromosom adalah sama dengan biangan kromosom sel induk. III To ensure that the genetic material in the daughter cells is the same as in the parent cell Untuk memastikan bahan genetik sel anak dalah sama dengan sel induk.

[Turn over CONFIDENTIAL

Page 121: BIO P1,P2 (2)

SULIT 4551/1

IV To contribute to the genetic variation in the daughter cells. Untuk menyumbangkan kepada variasi genetik dalam sel anak.

A I and II only C I , II and III only

B II and III only D I , III and IV only

16. Diagram 9 shows the correct proportion for the various classes of food in the food pyramid.

Rajah 9 menunjukkan nisbah yang betul bagi pelbagai kelas makanan dalam piramid makanan.

Which of the following shows the correct classes of food in the pyramid? Yang manakah di antara berikut menunjukkan kelas makanan yang betul dalam piramid makanan di atas?

1 2 3 4A Fats Proteins Carbohydrates Vitamins and

mineralsB Carbohydrates Vitamins and

mineralsProteins Fats

C Proteins Carbohydrates

Fats Vitamins andminerals

D Carbohydrates Fats Vitamins andminerals

Proteins

[Turn over CONFIDENTIAL

Diagram 9

Page 122: BIO P1,P2 (2)

SULIT 4551/1

17. Diagram 10 shows the structure of the human alimentary canal. Rajah 10 menunjukkan struktur salur alimentari manusia.

Diagram 10

Which organ produces the enzyme responsible for the breakdown of lipids ? Organ manakah merembeskan enzim yang bertanggungjawab untuk pemecahan lipid?

A. S only C R and SB. P and Q D Q only

18. Which adaptations help the villi to absorb nutrients efficiently? Yang manakah penyesuaian vilus untuk menyerap nutrien secara berkesan?

I Abundant in number Bilangan yang banyak

II Thin walls Dinding nipis III Having blood capillaries Mempunyai kapilari darahIV Lacteals to absorb fatty acids and glycerol Lakteal untuk menyerap asid lemak dan gliserol

A I and III onlyB II and IV onlyC I, II and III onlyD I, II, III and IV

19. When 0.4 g of groundnut is completely burnt, the temperature of 20 ml of water rise up from 30°C to 70°C. Calculate the energy value of the groundnut.

(Specific heat capacity of water is 4.2 Jg °C )Bila 0.4g kacang tanah terbakar dengan lengkap, suhu 20 ml air meningkat daripada 30°C kepada 70°C. Hitung nilai tenaga kacang tanah ?( Muatan haba tentu air ialah 4.2 Jg °C ).

A 1.4 kJg−¹ C 8.4 kJg−¹ B 3.4kJg-1 D 76.2 kJg−¹

[Turn over CONFIDENTIAL

Page 123: BIO P1,P2 (2)

SULIT 4551/1

20. Glucose + oxygen Carbon dioxide + water + 38 ATP

Glukosa + oksigen Karbon dioksida + air + 38 ATP

The above equation shows Persamaan di atas menunjukkan

A. aerobic respiration C condensation reaction

respirasi aerobik tindakbalas kondensasiB. anaerobic respiration D hydrolytic reaction

respirasi anaerobik tindakbalas hidrolitik

21. Which of the following organelle involves in the gaseous exchange in Ameoba sp? Antara organel berikut yang manakah terlibat dalam pertukaran gas dalam Ameoba sp ?

A Cell wall C Cell membrane Dinding sel Membran sel B Nucleus D Vacuole Nukleus Vakuol

22. Diagram 11 shows parts of the tracheal system of insect. Rajah 11 menunjukkan sebahagian daripada sistem trakea pada serangga.

Diagram 11

What process occurs at X during the gas exchange of the insect? Apakah proses yang berlaku di X semasa pertukaran gas bagi serangga tersebut?

A Diffusion C Facilitated diffusion Resapan Resapan berbantu

B Osmosis D Active transport Osmosis Pengangkutan aktif

[Turn over CONFIDENTIAL

bodytissue

tracheoles

Page 124: BIO P1,P2 (2)

SULIT 4551/1

23. Diagram 12 shows three different types of interaction between organisms. Rajah 12 menunjukkan tiga jenis interaksi di antara organisma..

Diagram 12Which of the following is true about the interactions K, L and M?Manakah di antara berikut benar tentang interaksi K , L dan M ?

K L MA Mutualism Commensalism ParasitismB Mutualism Parasitism SaprophytismC Saprophytism Commensalism MutualismD Parasitism Commensalism Mutualism

24. Which of the following chemical substance is used to kill or prevent the multiplication of microorganisms in the wound?

Yang manakah di antara sebatian kimia berikut, digunakan untuk membunuh atau mencegah pembiakan mikroorganisma dalam luka ?

A Antiseptic C Vaccine Antiseptik Vaksin

B Antibiotic D Disinfectant Antibiotik Disinfektan

25. Nitrates and phosphates from farmland that flow into a lake caused rapid growth of algae . What is described by the above situation ? Nitrat dan fosfat yang dialirkan dari ladang ke dalam tasik telah menyebabkan pertumbuhan alga yang mendadak. Apakah yang diterangkan oleh situasi di atas ?.

A Eutrophication C Fertilizer accumulation Eutrofikasi Pengumpulan baja

B Pesticide pollution D Colonisation Pencemaran pestisid Pengkolonian

[Turn over CONFIDENTIAL

Page 125: BIO P1,P2 (2)

SULIT 4551/1

26. Diagram 13 shows plant cells. Rajah 13 menunjukkan sel tumbuhan.Which of the following cell is the product of meiosis?

Sel yang manakah di antara berikut adalah produk pembahagian sel meiosis?

A B

C D

27 Diagram 14 shows a part of a mangrove plant.

Rajah 14 menunjukkan satu bahagian tumbuhan paya bakau.

What is structure S? Apakah struktur S?

A. Succulent leaves Daun sukulen B. Pneumatophores Pneumatofor C. Vivipary seeds Biji benih vivipariD. Prop roots

Akar jangkang

[Turn over CONFIDENTIAL

Diagram 14

Diagram 13

Page 126: BIO P1,P2 (2)

SULIT 4551/1

28. Diagram 15 shows a stage in the blood clotting mechanism. Rajah 15 menunjukkan satu peringkat di dalam mekanisme pembekuan darah.

Which of the following statement explains this stage? Antara pernyataan berikut, yang manakah menerangkan peringkat ini?

A

B

C

D

Thromboplastin converts prothrombin to thrombinTromboplastin menukarkan protrombin kepada trombinThrombin converts fibrinogen to meshwork of fibrin. Trombin menukarkan fibrinogen kepada jaringan fibrinPlatelets stimulate the formation of meshwork of fibrin. Platlet meransang pembentukan jaringan fibrin.Platelets release the thromboplastin to form meshwork of fibrin.Platlet membebaskan tromboplastin untuk membentuk jaringan fibrin.

29. Diagram 16 shows a cross-section through the carpel of a flower before fertilization.Rajah 16 menunjukkan keratan rentas melalui karpel bunga sebelum persenyawaan. Where are the position of male and female gametes before fertilization?Di manakah kedudukan gamet jantan dan betina sebelum persenyawaan ?

Male gamete Female gamete

A 1 5B 1 4C 2 4 D 3 5

[Turn over CONFIDENTIAL

Diagram 15

Diagram 16

Page 127: BIO P1,P2 (2)

SULIT 4551/1

30. Diagram 17 shows part of the placenta. Rajah 17 menunjukkan bahagian plasenta.

In which parts do the blood contain the most oxygen and nutrients?

Di bahagian manakah darah mempunyai kandungan oksigen dan nutrien yang tinggi?

A 1 and 3 C 2 and 3 B 1 and 4 D 2 and 4

31. Diagram 18 shows the structure of a nephron. Rajah 18 menunjukkan struktur nefron.

Which of the following activities cause X to be more permeable to water ? Yang manakah di antara aktiviti berikut menyebabkan X lebih telap kepada air ?

[Turn over CONFIDENTIAL

Diagram 17

Diagram 18

Page 128: BIO P1,P2 (2)

SULIT 4551/1

A P and R C Q and R

B P and S D Q and S

32. Diagram 19 shows the changes in the thickness of the uterus lining of a woman during her menstrual cycle. At which time is the woman most likely to be fertile?[Rajah 19 menunjukkan perubahan ketebalan lapisan uterus seorang wanita semasa kitar haid. Pada masa yang manakah wanita itu mengalami waktu paling subur?]

Diagram 19

33. The haemoglobin content of a pregnant mother is low. Which food should be taken to increase the haemoglobin content in her blood ?Kandungan haemoglobin seorang ibu mengandung adalah rendah. Makanan manakah yang perlu diambil untuk meningkatkan kandungan hemoglobin dalam darahnya?

A Spinach C Tomato Bayam Tomato

B Potatoes D Banana Kentang Pisang

[Turn over CONFIDENTIAL

P - Drinking a lot of water Minum air dengan banyakQ - Eating salty foods Makan makanan yang banyakR - Not exercising Tidak melakukan senamanS - Playing sports Bersukan

Page 129: BIO P1,P2 (2)

SULIT 4551/1

34. Diagram 20 shows a part of hind limp which consists of femur, tibia and fibula . Rajah 20 menunjukkan bahagian anggota belakang yang terdiri dari femur, tibia dan fibula.

Diagram 20

Which of this action cannot be done if the patella is dislocated? Manakah tindakan berikut tidak berlaku jika patela beralih tempat?

A Sitting down C Walking Duduk Berjalan

B Sleeping D Straightening the leg Tidur Meluruskan kaki 35. Diagram 21 shows the structures involved in reflex action. Rajah 21 menunjukkan struktur yang terlibat dalam tindakan refleks.

Which of the following shows the correct sequence for the above action?Antara berikut, manakah menunjukkan urutan yang betul bagi tindakan di atas?

A P Q R S C Q R S P

B P S R Q D Q S P R [Turn over

CONFIDENTIAL

Hot panPeriuk panas

Patela

Diagram 21

Page 130: BIO P1,P2 (2)

SULIT 4551/1

36. A farmer sprays all the mangoes in his farm with hormone X to ensure that all the mangoes ripen at the same time. What is hormone X?

Seorang petani menyembur semua buah manggadi ladang nya dengan hormon X bagi memastikan semua mangganya masak pada masa yang sama.

Apakah hormon X?

A Auxin C Cytokinin

B Ethylene D Gibberilin

37. Diagram 22 shows the structure of human brain. Rajah 22 menunjukkan struktur otak manusia.

What is X? Apakah X?

38. The following statements is about hormone X. Berikut adalah pernyatan tentang hormon X.

[Turn over CONFIDENTIAL

X

• Produced by corpus lutem and placenta[Dihasilkan oleh korpus luteum dan placenta]

• Promotes growth of endometrium and prevents menstruation.[Merangsang pertumbuhan endometrium dan menghalang haid]

Diagram 22

A CerebrumSerebrum

C Spinal cordSaraf tunjang

B CerebellumSerebelum

D Medula oblongataMedula oblongata

Page 131: BIO P1,P2 (2)

SULIT 4551/1

What is hormone X? [Apakah hormone X?]

A

Oestrogen C Luteinising hormone Estrogen Hormon pelutinan

B

Progesterone D Follicle stimulating hormoneProgesteron Hormon perangsang folikel

39. Which of the following is true when the osmotic pressure in the blood decreases? Manakah di antara berikut benar sekiranya tekanan osmosis darah berkurangan?

Secretion of ADHRembesan ADH

Reabsorption of water in kidney tubulesPenyerapan air oleh tubul ginjal

A IncreaseBertambah

IncreaseBertambah

B DecreaseBerkurang

DecreaseBerkurang

C DecreaseBerkurang

IncreaseBertambah

D IncreaseBertambah

DecreaseBerkurang

40. Diagram 23 shows the stages in the development of follicle in the ovary of human. Rajah 23 menunjukkan peringkat perkembangan folikel dalam ovari manusia.

What is the effect to the uterine wall when L developes into M? Apakah kesan kepada dinding uterus apabila L berkembang menjadi M?

[Turn over CONFIDENTIAL

ML Diagram 23

Page 132: BIO P1,P2 (2)

SULIT 4551/1

A It is repaired C It thickensIa diperbaiki Ia menebal

B It breaks down D Its thickness maintainsIa terurai Ketebalannya dikekalkan

41. Melissa who is a carrier for colour blindness married to Aron a normal colour vision.

What is the probability that their son is colour blind? Melisa adalah pembawa bagi buta warna berkahwin dengan Aron yang mempunyai penglihatan warna normal. Apakah kemungkinan anak lelaki mereka adalah buta warna ? A 0% C 50%

B 25% D 100%

42. A pair of fraternal twins are brought up by two different families and have the following characteristics.

Sepasang kembar seiras telah dibesarkan oleh dua keluarga yang berbeza dan mempunyai ciri seperti berikut.

Which factor causes the differences in the characteristics? Faktor yang manakah menyebabkan perbezaan ciri pada kembar itu?

A GeneticGenetik

C Gene mutationMutasi gen

B EnvironmentPersekitaran

D Chromosome mutationMutasi kromosom

43. Diagram 24 shows a red rose plant is crossed with a white rose plant. The F1 generations that are produced are two red rose plants and two white rose

plants. The allele for red rose plant, R is dominant to white rose plant, r. Rajah 24 menunjukkan pokok ros merah dikacukkan dengan pokok ros putih. Generasi F1 yang terhasil adalah dua pokok ros merah dan dua pokok ros putih. Alel untuk pokok ros merah, R adalah dominan kepada alel ros putih, r.

[Turn over CONFIDENTIAL

Nini : Fat and fair complexion Gemuk dan kulit cerah

Nina : Thin and slightly dark complexion Kurus dan kulit agak gelap

Page 133: BIO P1,P2 (2)

SULIT 4551/1

Diagram 24

What is the genotype of the parents?Apakah genotip bagi induk?

Red Rose White RoseA RR Rr

B Rr Rr

C Rr rr

D RR rr

[Turn over CONFIDENTIAL

ParentsPhenotype[Fenotip induk]

F1 generationPhenotype[Fenotip generasi F1]

Red Rose [Ros merah]

White Rose [Ros putih]

Red Rose [Ros merah]

White Rose [Ros Putih]

Red Rose [Ros merah]

White Rose [Ros Putih]

X

Page 134: BIO P1,P2 (2)

SULIT 4551/1

44. Diagram 25 shows ultrafiltration that occurs in the kidney. Rajah 25 menunjukkan ultraturasan yang berlaku dalam ginjal.

Diagram 25 What are the substances that can move across X ? Apakah bahan yang dapat merentasi X?

A

B

FibrinogenFibrinogenLeucocyte Leukosit

C

D

ErythrocyteEritrositAmino acidAmino asid

45. Diagram 26 shows a shirt with a blood stain before and after being washed with detergent containing enzyme. Rajah 26 menunjukkan baju dengan kesan darah sebelum dan selepas dibasuh

dengan pencuci mengandungi enzim.

Before After

Diagram 26

[Turn over CONFIDENTIAL

Afferent arterioleArteriol aferen

Efferent arterioleArteriol eferen

X

Blood stain

Bowman capsuleKapsul Bowman

Page 135: BIO P1,P2 (2)

SULIT 4551/1

Which is the most suitable enzyme and temperature to give the result shown?Yang manakah enzim dan suhu yang paling sesuai untuk menghasilkan keputusan seperti di atas?

46. Diagram 27 shows a pair of chromosomes in a cell of an organism. Rajah 27 menunjukkan sepasang kromosom dalam sel suatu organisma.

Diagram 27 What is X ? Apakah X ?

A Allele C Nucleotide B Gene D Chromosome

47. Diagram 28 shows the regulation of human body temperature. Rajah 28 menunjukkan pengawalan suhu badan manusia.

Diagram 28

[Turn over CONFIDENTIAL

Enzyme Temperature

A Lipase 37oC

B Protease 18oC

C Lipase 18oC

D Protease 37oC

X

Normal body temperatureSuhu badan normal

Body temperature increaseSuhu badan meningkat

Body temperature decreaseSuhu badan menurun

Temperature Regulation Centre Pusat kawalan suhu

Correction mechanismMekanisme pembetulan

Page 136: BIO P1,P2 (2)

SULIT 4551/1

Which of the following correction mechanism occur ?Antara berikut yang manakah mekanisme pembetulan yang berlaku?

l. Vasodilation III Vasoconstriction Pemvasodilatan Pemvasocerutanll. Erector muscle contract lV Decrease in metabolic rate Otot erektor mengecut] Kadar metabolisme menurun

A B

l and lll, ll and lll

CD

l and lVl, ll and lV

48. Diagram 29 shows the graphs of two types of variation . Rajah 29 menunjukkan graf untuk dua jenis variasi.

Plant Human Tumbuhan Manusia

Number of individualsBilangan individu

What type of variation shown in each population ? Apakah jenis variasi ditunjukkan dalam setiap populasi ?

Human

ManusiaPlant

TumbuhanA Continuous DiscontinuousB Continuous ContinuousC Discontinuous DiscontinuousD Discontinuous Continuous

49. The following food chain is found in a fresh water pond. Rantai makanan berikut terdapat dalam kolam air tawar.

Phytoplankton water fleas fish Fitoplankton kutu air ikan

Which of the following shows the relative amount of biological mass in the food

chain ? Manakah di antara berikut menunjukkan amaun jisim biologi relatif dalam rantai

[Turn over CONFIDENTIAL

Diagram 29

Page 137: BIO P1,P2 (2)

SULIT 4551/1

makanan ?

PhytoplanktonFitoplankton

Water fleaKutu air

FishIkan

A 5g 15g 60gB 30g 60g 15gC 60g 5g 30gD 60g 30g 5g

yang telah diberi suntikan vaksin sebanyak dua kali.

Which of the following will be the type of immunisation acquired by these two patients?Antara berikut yang manakah merupakan jenis keimunan yang diperoleh oleh kedua-dua pesakit?

A

B C

D

P QArtificial acquired active immunityKeimunan aktif buatan

Artificial acquired passive immunity Keimunan pasif buatan

Artificial acquired passive immunity Keimunan pasif buatan

Artificial acquired active immunityKeimunan aktif buatan

Natural acquired passive immunityKeimunan pasif semulajadi

Natural acquired active immunityKeimunan aktif semulajadi

Natural acquired active immunityKeimunan aktif semulajadi

Natural acquired passive immunityKeimunan aktif semulajadi

KERTAS SOALAN TAMAT.

[Turn over CONFIDENTIAL

Diagram 30

Page 138: BIO P1,P2 (2)

MARKING SCHEME

BIOLOGY 1 (4551/1)

SPM TRIAL EXAMINATION

2009

1. A 26. B2. B 27. C3. B 28. B4. A 29 D5. D 30. B6. D 31. D7. B 32. B8. D 33. A9. C 34. C10

.

C 35. C

11

.

D 36. B

12

.

B 37. B

13

.

B 38. B

14

.

A 39. A

15

.

C 40. C

16

.

B 41. C

17

.

D 42. B

18

.

D 43. C

19

.

C 44. D

20

.

A 45. D

21

.

C 46. A

22 A 47. B

Page 139: BIO P1,P2 (2)

23

.

C 48. D

24

.

A 49. D

25

.

A 50 A

Page 140: BIO P1,P2 (2)

Section ABahagian A

[60 marks][60 markah]

Answer all questions in this section.Jawab semua soalan dalam bahagian ini.

1 Diagram 1 shows the structure of a plasma membrane.Rajah 1 menunjukkan struktur membran plasma.

Diagram 1Rajah 1

(a) Name the parts labelled Q and R. Namakan bahagian yang berlabel Q dan R.

Q : ________________________________________________________

R : ________________________________________________________ [2 marks]

(b)(i) State the component of structure P. Nyatakan komponen struktur P.

___________________________________________________________ [1 mark ]

(ii) Explain the main function of P.

1(a)

1(b)(i)

[Lihat sebelah

2

For examiner’s use

RQ

P

Page 141: BIO P1,P2 (2)

Terangkan fungsi utama P.

___________________________________________________________

___________________________________________________________ [1 mark ]

(c) The plasma membrane is said to be semi-permeable. What is the meaning of ‘semi-permeable’?

Membran plasma dikatakan bersifat separa-telap. Apakah yang dimaksudkan dengan ’ separa-telap?’

______________________________________________________________

______________________________________________________________ [ 1 mark]

Diagram 1.2 Rajah 1.2

d) Graph in Diagram 1.2 shows the percentage of red blood cells that are burst or shrink when placed in salt solution of different concentration.

Graf dalam Rajah 1.2 menunjukkan peratus sel darah merah yang pecah atau mengecut apabila dimasukkan ke dalam larutan garam yang berbeza

kepekatan.

(i) Based on the graph given, state the concentration which is isotonic to blood plasma.

1(b)(ii)

1 (c)

[Lihat sebelah

3

Page 142: BIO P1,P2 (2)

Berdasarkan graf yang diberi, nyatakan kepekatan larutan yang isotonik terhadap plasma darah.

_________________________________________________________

(1 mark)

(ii) Explain your answer in (b)(ii). Terangkan jawapan anda dalam (b)(ii).

_________________________________________________________

(1 mark)

(iii) Comment on the osmotic pressure at Q. Berikan ulasan tentang tekanan osmosis di Q.

________________________________________________________________

________________________________________________________________

________________________________________________________________

________________________________________________________________ [ 2 marks]

(e) The concentration of ions inside root cells is up to 100 times greater than in the soil. Anyway, the ions are still transported into the cells by active transport.

Kepekatan ion di dalam sel akar adalah 100 kali lebih tinggi berbanding di dalam tanah. Walau bagaimanapun, ion-ion tersebut masih diangkut ke dalam sel secara pengangkutan aktif.

(i) Define active transport. Takrifkan pengangkutan aktif.

___________________________________________________________

___________________________________________________________ [ 1 mark]

1(d)(i)

1(d)(ii)

1(d)(iii)

1 (e)(i)

[Lihat sebelah

4

Page 143: BIO P1,P2 (2)

(ii) Explain what will happen to the uptake of the ions by root cells if the roots are immersed in a solution containing metabolic poisons such as cyanide.

Terangkan apa akan berlaku terhadap pengangkutan ion oleh oleh sel akar jika akar tersebut direndam di dalam larutan yang mengandungi racun metabolik seperti sianida.

___________________________________________________________

___________________________________________________________

___________________________________________________________

___________________________________________________________ [3 marks]

1(e)(ii)

TOTAL

[Lihat sebelah

5

Page 144: BIO P1,P2 (2)

2 Diagram 2.1 shows the biochemical processes involve molecule K, enzyme L and molecule M occur in organ X and organ Y. Rajah 2.1 menunjukkan proses biokimia yang melibatkan molekul K, enzim L dan molekul M yang berlaku di dalam organ X dan organ Y.

+ + + . . . . . .

G l y c o g e n

K

E n z y m e L

M

O r g a n X O r g a n Yh e p a t i c p o r t a l

v e i n

M u s c l e c e l l s

Diagram 2.1 Rajah 2.1

(a)(i) Name organ X and organ Y. Namakan organ X dan organ Y.

Organ X : ___________________________________________________

Organ Y : ___________________________________________________ [ 2 marks ]

(ii) Name molecule K, molecule M and enzyme L. Namakan molekul K, molekul M dan Enzim L.

Molecule K / molekul K : _______________________________________

Molecule M / molekul M :

_______________________________________

Enzyme L / enzim L : _______________________________________ [ 3 marks ]

(b) State two characteristics of enzyme L based on Diagram 2.1. Nyatakan dua ciri enzim L berdasarkan Rajah 2.1.

____________________________________________________________

____________________________________________________________

____________________________________________________________

2(a)(i)

2(a)(ii)

2 (b)

[Lihat sebelah

6

For examiner’s use

Page 145: BIO P1,P2 (2)

[ 2 marks ]

(c) Molecules M are transported from organ Y to muscle cells. Explain why molecule M is needed in muscle cells.

Molekul M diangkut dari Organ Y ke sel-sel otot. Terangkan kenapa molekul M diperlukan di dalam sel-sel otot.

____________________________________________________________

____________________________________________________________

____________________________________________________________

____________________________________________________________ [ 3 marks ]

(d) Explain the importance of forming glycogen. Terangkan kepentingan pembentukan glikogen.

____________________________________________________________

____________________________________________________________

____________________________________________________________ [ 2 marks ]

2 (c)

2 (d)

TOTAL

[Lihat sebelah

7

Page 146: BIO P1,P2 (2)

3 Diagram 3.1 shows three stages X, Y and Z in meiosis.

Rajah 3.1 menunjukkan tiga peringkat X, Y dan Z dalam meiosis.

Diagram 3.1Rajah 3.1

(a)(i) Name stages X and Y.

Namakan peringkat X dan Y.

X : ______________________________________________________

Y : ______________________________________________________

[2 marks]

(ii) State two differences between chromosomal behaviour at X and Y.

Nyatakan dua perbezaan perlakuan kromosom di X dan Y.

____________________________________________________________

____________________________________________________________

____________________________________________________________ [ 2 marks ]

(b) i) State the occurrence at Z.

Nyatakan kejadian yang berlaku di Z.

____________________________________________________________

___________________________________________________________

____________________________________________________________ [ 2 marks ]

3(a)(i)

(

3(a)(ii)

3(b)(i)

[Lihat sebelah

8

For examiner’s use

X Z Y

Soil

R S

T

Page 147: BIO P1,P2 (2)

ii) The chromosome number in somatic cell of this organism is 12. State the chromosome number in each of the daughter cell in Z. Give a reason for your answer.

Bilangan kromosom dalam sel soma bagi organisma ini ialah 12. Nyatakan bilangan kromosom dalam setiap sel anak Z. Berikan alasan anda.

______________________________________________________

______

____________________________________________________________

____________________________________________________________ [ 2 marks ]

(c) Diagram 3.2 shows the process of sperm formation in the human testis.

Rajah 3.2 menunjukkan proses pembentukan sperma di dalam testis manusia.

Diagram 3.2 Rajah 3.2

(i) Are cells A, cell B and cell C genetically identical? Explain.

Adakah sel A, sel B dan sel C seiras dari segi genetic? Terangkan.

____________________________________________________________

____________________________________________________________

____________________________________________________________

3(b)(ii)

3(c)(i)

[Lihat sebelah

9

Page 148: BIO P1,P2 (2)

____________________________________________________________[2 marks]

(ii) If Cell B undergoes an improper cell division, cell D might receive an extra chromosome. State the number of chromosomes in Cell D.

Jika Sel B melalui pembahagian sel yang tidak sempurna, sel D berkemungkinan menerima satu kromosom tambahan. Nyatakan bilangan kromosom di dalam sel D.

__________________________________________________________ [ 1 mark ]

(iii) If cell D fertilises with an ovum, the zygote formed might develop into an abnormal male. State the syndrome of the individual.

Jika sel D bersenyawa denngan ovum, zigot yang terbentuk akan berkembang menjadi lelaki yang abnormal. Nyatakan sindrom individu tersebut.

__________________________________________________________[ 1 mark ]

3(c)(ii)

3(c)(iii)

TOTAL

[Lihat sebelah

10

For examiner’s use

Page 149: BIO P1,P2 (2)

4 Diagram 4.1 shows the apparatus set up in an experiment to study the role of the vascular tissue in the transport of water in plants.

Rajah 4.1 menunjukkan susunan radas eksperimen untuk mengkaji peranan tisu vaskular dalam pengangkutan air di dalam tumbuhan.

Diagram 4.1

Rajah 4.1

(a) State the function of the eosin solution. Nyatakan fungsi larutan eosin.

___________________________________________________________ [ 1 mark ]

(b) The stem of the plant is cut across at XY and viewed under a microscope. A cross section of the stem is shown in Diagram 4.2.

Akar tumbuhan tersebut dikerat secara merentas pada XY dan diperhatikan di bawah mikroskop. Keratan rentas akar ditunjukkan dalam Rajah 4.2.

Diagram 4.2 Rajah 4.2

4(a)

4(b)

[Lihat sebelah

11

K

M

Page 150: BIO P1,P2 (2)

Name the parts labelled K and M. Namakan bahagian yang berlabel K dan M.

K : ___________________________________________________ M : ___________________________________________________ [ 2 marks ]

(c) Name the tissue which is responsible for transporting water and minerals ions from the roots to the upper parts of the plant.

Namakan tisu yang terlibat dalam pengangkutan air dan ion mineral dari akar ke bahagian atas tumbuhan.

____________________________________________________________ [ 1 mark ]

(d) If the root of the plant is cut across, draw and label the observation made.

Jika keratan rentas dibuat ke atas akar tumbuhan, lukis dan labelkan pemerhatian anda.

[ 3 marks ]

(e) Diagram 4.3 shows the effect of removing tissue M from the stem.

4(c)

4(d)

[Lihat sebelah

12

For examiner’s use

Page 151: BIO P1,P2 (2)

Diagram 4.3 Rajah 4.3

(i) State the type of transport involved in Diagram 4.3. Nyatakan jenis pengangkutan yang terlibat dalam Rajah 4.3.

___________________________________________________________ [ 1 mark ] (ii) Explain why does the part above the ring become swollen after two weeks.

Terangkan mengapa bahagian atas gelang membengkak selepas dua minggu.

___________________________________________________________

____________________________________________________________

___________________________________________________________

[2 marks]

(iii) Explain why have the leaves not wilted after two weeks. Terangkan mengapa daun-daun tidak layu selepas dua minggu.

___________________________________________________________

____________________________________________________________

___________________________________________________________

[2 marks]

4(e)(i)

4(e)(ii)

4(e)(iii)

TOTAL

[Lihat sebelah

13

Page 152: BIO P1,P2 (2)

5. Figure 5 shows the reflex arc that occurs when the finger is accidentally pricked with a needle.

Rajah 5 menunjukkan suatu arka refleks yang berlaku apabila jari tangan secara tidak sengaja dicucuki oleh sebatang jarum.

Diagram 5 Rajah 5

(a) Complete the above figure by drawing the appropriate neurones involved in the reflex action.

Lengkapkan rajah di atas, dengan melukis neuron yang terlibat di dalam tindak balas refleks.

[2 marks]

(b) Explain the transmission of impulse from one neurone to another neurone.

Terangkan pemindahan impuls dari satu neuron ke neuron yang berikutnya.

________________________________________________________________

________________________________________________________________

________________________________________________________________

________________________________________________________________

________________________________________________________________

________________________________________________________________ [ 4marks ]

5(a)

5(b)

5(c)

[Lihat sebelah

14

Page 153: BIO P1,P2 (2)

(c) Name the structures M and N. Namakan struktur M dan N.

M : ____________________________________________________

N : ____________________________________________________ [2 marks]

(d) Differentiate between the above reflex action with the voluntary action. Bezakan di antara tindakan refleks di atas dengan tindakan terkawal.

___________________________________________________________

___________________________________________________________

____________________________________________________________

[1 mark]

(e) State the importance of reflex action to us. Nyatakan kepentingan tindakan refleks kepada kita.

____________________________________________________________ [1

mark ](f) If efferent neurone is injured and damaged, predict what will happen to the person. Jika neuron eferen cedera dan rosak, ramalkan apa yang akan berlaku Kepada orang tersebut.

____________________________________________________________

____________________________________________________________ [1

mark ]

5(d)

5(e)

5(f)

TOTAL

Section BBahagian B

[40 marks]

[Lihat sebelah

15

Page 154: BIO P1,P2 (2)

Answer any two questions from this section.Jawab mana-mana dua soalan daripada bahagian ini

6(a) Figure 6.1 shows movement activities in a human.Rajah 6.1 menunjukkan aktiviti pergerakan pada manusia.

.Based on Figure 6.1(i) and Figure 6.1(ii), explain how the above movement takes place which involves muscles, tendons, bones, ligaments and joints

Berdasarkan Rajah 6.1(i) dan Rajah 6.1(ii), terangkan bagaimana pergerakan di atas berlaku yang melibatkan otot, tendon , tulang, ligamen dan sendi.

[10 marks] (b)(i) By giving one example of woody plant and and non-woody, explain how

the support system in woody plants differs from that of non-woody plants.Dengan menyatakan contoh, terangkan bagaimana sistem sokongan pada tumbuhan berkayu berbeza daripada tumbuhan tidak berkayu.

[10 marks]

7(a) Diagram 7.1 shows how the respiratory gases are transported in the human body.Rajah 7.1 menunjukkan bagaimana gas respirasi diangkut dalam badan manusia.

[Lihat sebelah

16

(i) (ii)Diagram 6.1

Page 155: BIO P1,P2 (2)

Diagram 7.1

(i) Based on Diagram 7.1, explain how the transport of oxygen and carbon dioxide takes place in the body cellsBerdasarkan Rajah 7.1, terangkan bagaimanakah pengangkutan oksigen dan karbon dioksida berlaku di dalam sel-sel badan.

[6 marks]

(ii) Describe the adaptations of the alveolus for gaseous exchange.Terangkan penyesuaian pada alveolus yang membolehkannya utuk melakukan proses pertukaran gas.

[4 marks]

(b) The shaded area of the graph in Diagram 7.2 shows the intake of oxygen by an athlete before, during and after running for five minutes.Kawasan yang berlorek pada graf dalam Rajah 7.2 menunjukkan pengambilan oksigen oleh seorang atlet sebelum, semasa dan selepas

[Lihat sebelah

17

Page 156: BIO P1,P2 (2)

berlari selama 5 minit.

Diagram 7.2

Based on the graph, explain how an oxygen debt is built up when an athlete is running and how it is settled after he stops running.Berdasarkan graf, terangkan bagaimana hutang oksigen terhasil semasa atlet itu berlari dan bagaimana ia diselesaikan selepas beliau berlari.

[10 marks]

8(a) Mr Nick has group A blood while his wife has group B blood. The group of their son is O. Explain how this happen.

[Lihat sebelah

18

Page 157: BIO P1,P2 (2)

Mr Nick mempunyai kumpulan darah A manakala isterinya mempunyai kumpulan darah B. Kumpulan darah anak lelaki mereka ialah O.Terangkan bagaimana ini boleh berlaku.

[10 marks]

(b) Nowadays, the DNA fingerprinting technique has replaced the common fingerprinting technique in criminal investigations.

Pada masa kini, teknik cap jari DNA telah menggantikan teknik cap jari biasa dalam penyiasatan jenayah.

(i) Explain how DNA fingerprinting is carried out.Terangkan bagaimana cap jari DNA dilakukan

[4 marks]

(ii) Based on the given statement, state your opinion an the advantages and disadvantages of DNA fingerprinting.

Berdasarkan pernyataan yang diberikan, nyatakan pendapat kamu tentang kebaikan dan keburukan cap jari DNA.

[6 marks]

9(a) Diagram 9 shows a mangrove swamp.Rajah 9 menunjukkan kawasan paya bakau.

[Lihat sebelah

19

Page 158: BIO P1,P2 (2)

Diagram 9

(i) Explain why most plants cannot colonise and grow in the swamps.Terangkan mengapa kebanyakan tumbuhan tidak boleh hidup dan tumbuh di kawasan paya bakau.

[5 marks]

(ii) Explain how the mangrove trees adapt themselves to the harsh living conditions.Terangkan bagaimana pokok bakau ini menyesuaikan diri dengan keadaan hidup yang sukar.

[5 marks]

(b) Development that is not planned and managed properly has brought negative effects to the ecosystem such as land erosion, flash flood, landslides, global warming, thinning of the ozone layer, climate change and the extinction of certain species.Pembangunan yang tidak terancang dan terurus dengan teliti boleh membawa kesan negatif kepada ekosistem seperti hakisan tanah, banjir kilat, tanah runtuh, pamanasan global, penipisan lapisan ozon, perubahan iklim dan kepupusan spesis tertentu.

Based on the above statement, describe the effects of unplanned development and improper management of the ecosystem.Berdasarkan kenyataan di atas, terangkan kesan pembangunan yang tidak terancang dan terurus dengan teliti kepada ekosisitem.

[10 marks]

[Lihat sebelah

20

Page 159: BIO P1,P2 (2)

MARKING SCHEME

BIOLOGY 2 (4551/2)

SPM TRIAL EXAMINATION

2009

No. Marking Criteria Mark1(a)

1(b)(i)

(ii)

1(c)

1 (d)(i)

1(d)(ii)

Able to name the parts labelled Q and R.

Sample answer :

Q : Carrier proteinR : Channel protein / pore protein

Able to state the component of structure P.

Sample answer :

It is composed of two layers of phospholipids

Able to explain the main function of P.

Sample answer :

Acts as a barrier between the internal and external environment of the cell // Allows only specific molecules to pass through it // provide the structural basis for all cell membrane. Able to give the meaning of ‘semi-permeable’.

Sample answer :

A semi-permeable plasma membrane is a membrane that allows only certain substances to move freely across it. Able to state the concentration which is isotonic toblood plasma. Sample answer :

0.45 g/100 cm3

Able to explain the answer in (d)(i).

11

1

1

1

1

2

2

1

Page 160: BIO P1,P2 (2)

1(d)(iii)

1(e)(i)

(ii)

SampleAnswer :

Both percentage of haemolysis of red blood cells and percentage of crenation of red blood cells are zero (0%).

Able to comment on the osmotic pressure at Q.

SampleAnswer :

F : The osmotic pressure inside the red blood cells is equivalent to its environment.P2 : Amount of water moving in and out of the cells are the same,P3 : therefore the size and structure of the red blood cells does not change. ( F + Any P2/P3 )

Able to define active transport.

Sample answer :

Active transport is a movement of substances / molecules / ions against the concentration gradient / from low to high concentration across the plasma membrane with the help of carrier protein and energy / ATP.

Able to explain what will happen to the uptake of the ions by root cells if the roots are immersed in a solution containing metabolic poisons such as cyanide.

Sample answer :

P1 – there is no uptake of ions by root cellsP2 – metabolic poisons kill/ damaged the (root) cellsP3 – no energy/ ATP is producedP4 – active transport does not occur (Any three)

1

1

1

1

3

TOTAL

4

4

13 marks

Page 161: BIO P1,P2 (2)

2(a)(i)

(ii)

(b)

(c)

(d)

Able to name organ X and organ Y.

Sample answer :

Organ X : Ileum // small intestineOrgan Y : Liver

Able to name molecule K, molecule M and enzyme L.Sample answer :

Molecule K : Starch Molecule M : GlucoseEnzyme L : (Pancreatic) Amylase

Able to state two characteristics of enzyme L based on Diagram 2.1

Sample answer :

1. Enzyme remains unchanged at the end of the reaction (and can be used again).

2. Enzyme is substrate specific / reaction is very specific

Able to explain why molecule M is needed in muscle cells.

Sample answer :

Pt. 1 Molecule M / glucose is the substrate for respiration Pt. 2 As the muscle cells contract and relax, energy is needed for activitiesPt. 3 therefore, molecule M is needed in muscle cells to provide energy from respiration process.

Able to explain the importance of forming glycogen.

Sample answer :

Pt.1 : Glycogen is the main reserve of carbohydrates in animals

11

111

1

1

1

1

1

1

5

2

3

Page 162: BIO P1,P2 (2)

Pt. 2 It can be converted back to glucose when energy is needed from respiration process

1

TOTAL

2

12 marks

3(a(i))

(ii)

(b)(i)

Able to name stage X and Y.

Sample answer :

X : Prophase IY : Metaphase I

Able to Able to state two differences between chromosomal behaviour at X and Y.

Sample Answer:

11

1

1

1

11

2

2

2

Page 163: BIO P1,P2 (2)

(ii)

(c)(i)

(ii)

(iii)

Prophase IMetaphase I

(Paired homologous chromosomes) are arranged randomly.(Paired homologous chromosomes) are arranged on the metaphase plate / equatorial plane.

Spindle fibre does not hold on the centromere of the chromosomes .Spindle fibre holds on the centromere of the chromosomes.

(The homologous chromosomes paired and) crossing over take place.(The homologous chromosomes paired) crossing over does not take place.

( Any 2 )

Able to state the occurrence at Z.

Sample Answer:

P1 : Four daughter cells formed P2 : Each daughter cell has two chromosomes / haploid / n

Able to state the chromosome number in each of the daughter cell in Z and able to give reason.

Sample answer :

1

1

1

1

1

1

1

TOTAL

2

2

2

12 marks

Page 164: BIO P1,P2 (2)

P1 : 6 (chromosomes). P2 :(During meiosis) the daughter cell receives half the number of chromosome from the parent cell / 2n // Daughter cell haploid / n, parent cell diploid / 2n

Able to state either cell A, cell B and cell C are genetically identical and explain.

Sample answer :

F : Cell A is similar to cell B but is different from cell C. P : Cell A and cell B are products of mitosis whereas cell C is a product of meiosis. Able to state the number of chromosome in Cell if Cell B undergoes an improper cell division.

Sample answer :

24 (chromosomes) Able to state the syndrome of the individual.Sample answer :

Down’s syndrome // Klinefelter’s syndrome

4(a)

4(b)

4(c)

Able to state the function of the eosin solution.

Sample answer :

To stain the xylem (vessels) (with red dye)

Able to name the parts labelled K and M. Sample answer :

K : XylemM : Phloem

Able to name the tissue which is responsible for transporting water and mineral ions from the roots to the upper parts of the plant. Sample answer :

1

11

1

2

Page 165: BIO P1,P2 (2)

4(d)

(e)(i)

(ii)

(iii)

Xylem

Able to draw and label the observation of the root cut across.

Sample answer :

Drawing – 1 mAny 2 labels – 2 m

Able to state the type of transport involved in Diagram 4.3.

Sample answer :

Translocation

Able to explain why does the part above the ring become swollen after two weeks.

Sample answer :

F : The products of photosynthesis cannot be transported to the parts below the ring

P : as tissue M / phloem is removed

Able to explain why have the leaves not wilted after two weeks.

Sample answer :

F : Water can still be transported to the leavesP : as tissue K / xylem is not removed from the stem

1

12

1

11

11

TOTAL

1

3

5

12 marks

Xylem

PhloemPericycle

Cortex // ground tissue

Page 166: BIO P1,P2 (2)

5(a)

5(b)

5(c)

Able to complete the drawing the appropriate neurons involved in the reflex action.

Sample answer :

3 neurones – 2 m 2 neurones - 1 m

Able to explain the transmission of impulse from one neurone to another neurone.

Sample answer :

Pt..1 When an impulses arrives in the axon terminalPt. 2 it stimulates (synaptic) vesicles to move towards and bind with the presynaptic membranePt. 3 The vesicles fuse / release the neurotransmitter into

the synapse Pt. 4 The neurotransmitter molecules across the synapse

to the dendrite of another neurone Pt. 5 Stimulated to trigger a new impulses which travels

along the neurone ( Max 4 )

Able to name the structure M and N.

Sample answer :

M : Sensory reseptor // finger tipN : Effector // muscles tissues

2

1

1

1

1

1

11

2

4

2

Page 167: BIO P1,P2 (2)

5(d)

5(e)

5 (f)

Able to differentiate the reflex action with the voluntary action.

Sample answer :

The reflex action is governed by the spinal chord whereas the voluntary action is governed by the cerebrum.

Able to state the importance of reflex action to us. Sample answer :

To protect the body against injuries

Able to predict the effect on O if it is injured or damaged.

Sample answer:

1. The nerve impuls will be sent from afferent neurone to the effector2. The effector / muscles will not contract3. The hand will not be removed immediately from the needle.

(Any one )

1

1

11

1

TOTAL

1

1

1

11 marks

Page 168: BIO P1,P2 (2)

6(a)

(b)

Able to relate the tissues involved in producing the running movement

Sample Answer:

Pl- Tendons, ligaments, bones, muscles and joints are important features in a movement,P2- Tendons connect muscles to bonesP3- Tendons are strong and non elasticP4- Force is transferred to bones through tendons.P5- Movement at the joint is possible with the aid of ligaments.P6- Ligaments connect two bones together P7-to give support and strength to the joint.P8- Ligaments are strong and elastic.P9- The quadriceps / extensor muscles contract while the biceps femoris muscles relax and the leg is straightened.P10- The biceps femoris muscles contract while the quadriceps / extensor muscles relax and the leg is bent.P11- Calf muscles contract to lift up the heels.P12-Feet push downward and backwardP13-Repeated contraction and relaxation of muscles result in the running movement.

MAXIMUM: 10 marks

Able to give example and explain how the support system in woody plants differs from that of non-woody plants.

Examples – 2 marks , Facts – 8 marks

Sample answers:

Non-woody plants (herbaceous plants)Example: Balsam plant/ any suitable answer

P1: (Support in herbaceous plants is) provided by the turgidity of the parenchyma / collenchyma cells

1

11111

11

1

111

1

1

1

10

Page 169: BIO P1,P2 (2)

P2: (When there is enough warm in the ground). the cells take in water by osmosis and become turgid.P3: The turgor pressure of the fluids in the vacuoles pushes the cell contents / plasma membrane against the cell wall P4: creating support for it stem/ roots /leavesP5: The thin thickening die cell walls with cellulose / collenchyma cells gives support to herbacous plantsWoody plants :Example : Rambutan tree/ hibiscus/ any suitable example

P6: Woody plants have specialised tissues/ sclerenchyma tissues/ xylem vessels / tracheids. to give them support; P7: These tissues have cellulose walls which have deposits of lignin for added strength. P8: Sclerenchyma cells have very thick walls (which do not allow water to pass through).P9: (These cells are dead cells and) their function is to pro-vide support for the plant.P10: Xylem vessels have thick walls of lignin which are deposited during the plant's secondary growth.P11: The lignified xylem vessels form the woody tissues of the stem.P12: This makes the plant stronger and also provides support for the plant.P13: Tracheids are also dead cells with thick walls and very small diameters.P14: They are found with the xylem vessels and together they support the plants.

MAXIMUM: 10 marks

1

11

1

1

1

1

1

1

1

1

1

1

10

20 marks

Page 170: BIO P1,P2 (2)

TOTAL

7(a) (i) Able to explain how the transport of oxygen and carbon dioxide takes place in the body cells

Sample answers:

P1: The blood circulatory system transport oxygen from the alveoli to the body cells. P2: Oxygen combines with the haemoglobin in the red blood cells P3: to form oxyhaemoglobin (which is unstable.)P4: Oxygen is carried (in form of oxyhaemoglobin) to the tissues (which have a low partial pressure of oxygen.)P5: The (unstable) oxyhaemoglobin breaks down into oxygen and haemoglobin again.P6: Oxygen (molecules are) transferred to the body cellsP7: Carbon dioxide binds (itself) to the haemoglobin P8: (and is) transported in the form of carbaminohaemoglobin.P9: Carbon dioxide is (also) transported as dissolved carbon dioxide (in the blood plasma.)P10: Most of carbon dioxide is carried as bicarbonate ions (dissolved in the blood plasma.)P11: When the blood carrying carbon dioxide reaches the body cells, the carbon dioxide diffuses into the blood plasma and combines with the red blood cells.P12:Carbon dioxide reacts with water to form carbonic acid.P13:Carbonic anhydrase in the red blood cells catalyse the formation of carbonic acid.P14: The carbonic acid then dissociates into a hydrogen ions and bicarbonate ions.

MAXIMUM: 6 marks

(ii) Able to describe the adaptations of the alveolus for gaseous exchange

Sample answer:

F1: The millions of alveoli P1: provide a large surface area for gaseous exchange.F2: The walls of the alveoli are moist P2: and this allows respiratory gases to dissolve easily to them.F3: The walls of the alveoli are very thin (one-cell thick)P3: forquick / easy diffusion of gases.F4: The alveoli are richly supplied with blood capillaries

1

111

1

1111

1

1

11

1

6

Page 171: BIO P1,P2 (2)

(b)

P4: to increase the rate of diffusion / the rate of the transportation of gases

MAXIMUM: 4 marks

Able to explain how an oxygen debt is built up when an athlete is running and how it is settled after he stops running.

Sample answer:

P1: During a vigorous exercise /running, the breathing rate is increased.P2: This is to supply more oxygen (quickly to the muscles)P3:for rapid muscular contraction).P4: However, the supply of oxygen to muscles is still insufficient P5: and the muscles have to carry out anaerobic respiration (to release energy).P6: The glucose is converted into lactic acid,P7: with only a limited amount of energy being producedP8: An oxygen debt builds up in the body as shown in the graphP9: High levels of lactic acid in the musclesP10: cause them to ache.P11: After running, the athlete breathes more rapidly / deeply than normal for 20 minutes (shown in the graph)P12: There is a recovery period (from the 10th minute until the 20th minute) P13:when oxygen is paid back (during aerobic respiration)P14: About 1/6 lactic acid is oxidised to carbon dioxide, water and energy.

MAXIMUM: 10 marks

4

10

20 marks

Page 172: BIO P1,P2 (2)

TOTAL

8(a)

(b)

Able to explain how the inheritance happen

Answer :

P1: The situation involved is monohybrid inheritance. P2: The genotype of blood group A can be IAIA /1A10

P3: while the genotype of blood group B can be IBIB or IBIO.P4: Blood group 0 has a genotype, IOIO (while the genotype of blood group AB is IAIB.P5: Alleles 1A and IB are codominant P6: IO allele is recessive.P7: Mr. Nick is heterozygous dominant/IAIO (for his blood group A)P8: while his wife is heterozygous dominant/ IBI0 (for blood group B)P9: Mr. Nick and his wife produce haploid gametes/sperm/ovum (as a r e s u l t o f m e i o s i s )P10: Mr. Nick produces (gametes with) genotypes IA /IO

P11: (while) his wife (will) produce (gametes with) genotypes 1A/ lO

P12: The gamete (IO) of Mr. Nick fuses with his wife's gamete (10)P13: to produce a zygote with genotype I°Io.P14: (Thus, they will) produce an offspring with blood group 0.

MAXIMUM: 10 marks

(i) Able to explain how DNA fingerprinting is carried out.

Answer:

P1: Tissue samples are taken from the scene of a crime and DNA is extracted.P2: An enzyme breaks down the DNA into fragments.

P3: The DNA fragments are classified according to size.

P4: An alkali is added to separate the double-stranded D N A

i n t o s i n g l e s t r a n d s .

P 5 : Each single strand is laid on a nylon membrane and

1111

1111

1

11111

1

1

1

1

1

1

10

Page 173: BIO P1,P2 (2)

radioactive matter is added to it. A banding pattern appears.

P6: An X-ray film is produced and the positions of black bands

are compared with the part of DNA treated with radioactive

matter.

MAXIMUM: 4 marks

(ii) Able to state the advantages and disadvantages of

DNA fingerprinting

Sample answer:

Advantages:

P1: DNA fingerprinting is more accurate than common

fingerprinting as no two people have the same DNA

fingerprints.

P2: DNA fingerprinting is more efficient than blood-type

identification because many people have the s a m e

b l o o d t y p e

P3: DNA fingerprinting requires only a small amount of DNA to

obtain a highly accurate result

P4: DNA samples last longer than fingerprints.

P5: Mixed DNA samples can still be used.

P6: DNA evidence is harder to clean up compared to

fingerprints.

Disadvantages:

P7: DNA samples may be degraded by adding

chemicals, and this will affect the accuracy of the technique.

P8: Human errors are possible when different procedures and

standards are used in DNA fingerprinting.

MAXIMUM: 6 marks

1

1

1

1

1

1

1

1

TOTAL

4

6

20 marks

Page 174: BIO P1,P2 (2)

9(a) (i) Able to explain why most plants cannot colonise and grow in the swamps.

Sample answer:

P1: The ground is too soft and unable to support plants,P2: The water-logged / muddy swamps provide very little oxygen for root respiration.P3: The swamp water has a high concentration of salt and is hypertonic.P4: The plants growing in swamp will have the problem of dehydration.P5: Seeds that fall into the muddy swamp will die of dehydration / insufficiency of oxygen.P6: The swamp is exposed to strong sunlight and intense heat. P7: As a result, the plants growing there will lose water very fast by transpiration.

MAXIMUM: 5 marks

(ii) Able to explain how the mangrove trees adapt themselves to the harsh living conditions

Sample answer:

P1: Root system which is highly branched and spreads over a big area to give good support to the plants.P2: Pneumatophores (breathing roots) which grow protruding upwards above the ground.P3: The plant cells have high concentration of cell sap. P4: Hence, the cells are able to withstand the high salt content of the swamp.P5: Excess salt is eliminated through hydatodes found at the lower epidermis of leaves.P6: Viviparous seeds which germinate while still attached to the parent plant.P7: The long radical produced will let the seedling stick into the ground and not submerge or drift away.P8:Thick cuticle and sunken stomata which help to reduce the rate of transpiration.

11

1

1

1

11

11

11

5

5

Page 175: BIO P1,P2 (2)

(b)

MAXIMUM: 5 marks

Able to describe the effects of unplanned development and improper management of the ecosystem.

P1: The leave canopy in the forest protects the soil from

excess rain water.

P2: When the forest is cleared, the soil is exposed to rain

(water) / wind.

P3: this will cause soil erosion

P4:The soil that is exposed to wind will be blown to

another area,

P5: while soil that is exposed to rain water will be eroded

and deposited at the bottom of the river / pond /lake.

P6:The soil at the hill slopes can (also) be washed away by

heavy rain water

P7: resulting in land slides.

P8: (The deposited soil will) cause the water level to increase

rapidly when it rains and

P9: this will in turn cause flash floods.

P10:Wild life species will also be threatened

P11: when their habitat is destroyed.

P12: Global warming will occur

P13:due to an increase in the Earth's temperature,

P14:which is caused by excess emissions of carbon

dioxide/ methane/ CFC /nitrogen dioxide (into the

atmosphere).

P15:These gases trap the heat that is reflected by the Earth.

P16:The thinning of the ozone layer occurs

P17: when the ozone layer (that protects the Earth from

ultraviolet radiation) is destroyed by chlorofluorocarbons

(CFC).

MAXIMUM: 10 marks

1

1

11

1

1

1

1

1

11111

111

10